You are on page 1of 70

Total Marks : 200

Online Prelims TEST - 12 (SUBJECT WISE)


( InsightsIAS Mock Test Series for UPSC Preliminary Exam 2020 )

1 Consider the following statements:


1. Tiger, man, cat and dog can be considered as examples for stenothermal animals.
2. Seals can be considered as example for cryophilic animals.
3. The green crab is an example of a euryhaline invertebrate.

Which of the statements given above is/are correct?


A. 1 and 2 only
B. 1 only
C. 2 and 3 only
D. 1, 2 and 3

Correct Answer : C

Answer Justification :

Eurythermal animals are those who can tolerate a wide range of temperatures. They are able to
function at a wide range of body temperatures. Eurythermal animals show less sensitivity to
temperature or they show comparatively low temperature sensitivity. Examples of eurythermal
animals are desert pupfish, green crabs, tiger, man, cat, dog etc. Hence, statement 1 is
incorrect.

Stenothermal animals are those who are able to tolerate a narrow range of temperatures.
Marine and soil organisms are mostly stenothermal. They are able to survive at certain
temperatures.

And the temperature of the stenothermal animals varies from species to species. Stenothermal
animals are main two types namely thermophilic animals and cryophilic animals.

Thermophilic animals can live only in higher temperatures. Examples of thermophilic animals
include reptiles, insect species, etc.

Cryophilic animals can live only at low temperatures. Seals can be considered as example for
cryophilic animals. Hence, statement 2 is correct.

Euryhaline organisms are able to adapt to a wide range of salinities. An example of a


euryhaline fish is the molly (Poecilia sphenops) which can live in fresh water, brackish water, or salt
water. The green crab (Carcinus maenas) is an example of a euryhaline invertebrate that
can live in salt and brackish water. Hence, statement 3 is correct.

Euryhaline organisms are commonly found in habitats such as estuaries and tide pools where the
salinity changes regularly. However, some organisms are euryhaline because their life
cycle involves migration between freshwater and marine environments, as is the case
with salmon and eels.

https://en.wikipedia.org/wiki/Euryhaline

https://www.differencebetween.com/difference-between-eurythermal-and-vs-stenothermal-animals/

1
Total Marks : 200
Online Prelims TEST - 12 (SUBJECT WISE)
( InsightsIAS Mock Test Series for UPSC Preliminary Exam 2020 )

2 Which of the following type of algae that inhabit the sea is/are likely to be found in the deepest waters?

A. Red algae
B. Green algae
C. Brown algae
D. None

Correct Answer : A

Answer Justification :

Red algae are able to grow in depth of seas as compared to green algae because of the presence
of more quantity of pigment phycoerythrin in red algae growing at depth. Phycoerythrin can
absorb blue- green wavelength of visible spectrum of light that can reach the maximum
depth of water and help in photosynthesizing those red algae.

Hence, option (a) is correct.

3 Consider the following statements:


1. Aestivation is the type of summer sleep, performed by cold-blooded animals.
2. Hibernation is the type of winter sleep, performed by the warm as well as cold-blooded animals.

Which of the statements given above is/are correct?


A. 1 only
B. 2 only
C. Both 1 and 2
D. Neither 1 nor 2

Correct Answer : C

Answer Justification :

The state of inactivity and a low metabolic process performed by the animals during the
winters is known as Hibernation. It is also known as winter sleep. It may be performed by both
the warm and cold-blooded animals. Examples include bats, birds, mammals, insects, etc

Hence, statement 2 is correct.

On the contrary, when animals take rest in shady and moist place during summer, it is
called Aestivation or Estivation. Aestivation is also known as summer sleep. It is performed by
cold-blooded animals. Examples include bees, snails, earthworms, salamanders, frogs,
earthworms, crocodiles, tortoise, etc

Hence, statement 1 is correct.

2
Total Marks : 200
Online Prelims TEST - 12 (SUBJECT WISE)
( InsightsIAS Mock Test Series for UPSC Preliminary Exam 2020 )

4 Consider the following birds:


1. Amur Falcon
2. Greater Flamingo
3. Bluethroat
4. Bar Headed Goose

Which of the birds above given do migrate during winter season?


A. 1, 2 and 3 only
B. 2, 3 and 4 only
C. 1 and 4 only
D. 1, 2, 3 and 4

Correct Answer : D

Answer Justification :

Amur falcons are small raptor that breed in Siberia and migrate to Southern Africa. During
migration in large flocks across North East India and over the Arabian sea, they feed around
Doyang Lake in Nagaland in the evening or early in the morning during the winter season.
Hence, statement 1 is correct.

Greater Flamingo is the largest of all species of the flamingo family, found in the Indian
subcontinent. The migratory bird found in parts of Nal Sarovar Bird Sanctuary, Khijadiya
Bird Sanctuary, Flamingo City and in the Thol Bird Sanctuary of Gujarat during entire
winter season. Hence, statement 2 is correct.

Bluethroat is a small brightly colored passerine bird from thrush family and winters in north
Africa and the Indian subcontinent. Keoladeo National Park of Bharatpur in Rajasthan is
one of the best place to spot migratory birds coming to India in winter season. Hence,
statement 3 is correct.

Bar Headed Goose is the highest flying bird in the world and migrate to India every in
winter season. The bar headed migrates over the Himalayas and spend their winter in Assam to
Tamil Nadu states of India. Hence, statement 4 is correct.

http://www.walkthroughindia.com/wildlife/10-beautiful-migratory-birds-coming-india/

5 Which of the following reasons can be attributed for rarely finding very small animals in Polar
Regions?
1. Very small animals have a smaller surface area relative to their volume.
2. Very small animals tend to lose body heat very fast when it is cold outside.
3. Very small animals have to expend much energy to generate body heat through metabolism.

Select the correct answer using the code given below:


A. 1 and 3 only
B. 2 only
C. 2 and 3 only
3
Total Marks : 200
Online Prelims TEST - 12 (SUBJECT WISE)
( InsightsIAS Mock Test Series for UPSC Preliminary Exam 2020 )

D. 1, 2 and 3

Correct Answer : C

Answer Justification :

Thermoregulation is energetically expensive for many organisms. This is particularly true for
small animals like shrews and humming birds. Heat loss or heat gain is a function of surface
area. Since small animals have a larger surface area relative to their volume, they tend to
lose body heat very fast when it is cold outside. Hence, statement 1 is incorrect and
statement 2 is correct.

They have to expend much energy to generate body heat through metabolism. Hence,
statement 3 is correct.

This is the main reason why very small animals are rarely found in polar regions.

6 How do body acclimatized itself to the altitude sickness in higher altitude during mountain climbing?
1. Decreasing red blood cell production.
2. Increasing the binding capacity of hemoglobin.
3. Increasing breathing rate

Select the correct answer using the code given below:


A. 1 and 2 only
B. 3 only
C. 1 and 3 only
D. 1, 2 and 3

Correct Answer : B

Answer Justification :

Altitude sickness means the physical distress from difficulty adjusting to lower oxygen pressure at
high altitude. Its symptoms include nausea, fatigue and
heart palpitations.

But, gradually you get acclimatized and stop experiencing altitude sickness. The body compensates
low oxygen availability by increasing red blood cell production, decreasing the binding
capacity of hemoglobin and by increasing breathing rate.

Hence, both statement 1 and statement 2 are incorrect. However, statement 3 is correct.

7 Which of the following organisms do breed only once in lifetime?


1. Elephant
2. Bamboo

4
Total Marks : 200
Online Prelims TEST - 12 (SUBJECT WISE)
( InsightsIAS Mock Test Series for UPSC Preliminary Exam 2020 )

3. Pacific Salmon fish

Select the correct answer using the code given below:


A. 3 only
B. 1 only
C. 2 and 3 only
D. 1, 2 and 3

Correct Answer : C

Answer Justification :

Under a particular set of selection pressures, organisms evolve towards the most efficient
reproductive strategy. Some organisms breed only once in their lifetime (Pacific salmon fish,
bamboo) while others breed many times during their lifetime (most birds and mammals).
Some produce a large number of small-sized offspring (Oysters, pelagic fishes) while others produce
a small number of large-sized offspring (birds, mammals).

Unlike the bigger, more abundant savannah elephants – which start breeding from the age of 12 –
female forest elephants begin breeding only at 23. They then only give birth only once every five to
six years.

Hence, option (c) is correct.


https://www.newscientist.com/article/2103783-slow-to-breed-elephant-hurtles-towards-extinction/#i
xzz62RQpxAna

8 With reference to the Green Mahanadi Mission, consider the following statements:
1. It has been launched by Odisha government.
2. The objective of the mission is to recharge and revitalize the groundwater reserve along Mahanadi
banks.
3. Plantation within one km radius of the banks on both sides would primarily include fruit-bearing
trees.

Which of the statements given above is/are correct?


A. 2 only
B. 1 and 3 only
C. 2 and 3 only
D. 1, 2 and 3

Correct Answer : D

Answer Justification :

Odisha government launched the ‘Green Mahanadi Mission’, a mega plantation drive along the river
basin. Hence, statement 1 is correct.

5
Total Marks : 200
Online Prelims TEST - 12 (SUBJECT WISE)
( InsightsIAS Mock Test Series for UPSC Preliminary Exam 2020 )

The objective of the mission is to recharge and revitalize the groundwater reserve along
Mahanadi banks. Hence, statement 2 is correct.

Plantation within one km radius of the banks on both sides would primarily include fruit-
bearing trees like mango, jackfruit, jamun etc. which will have multiple benefits such as
ground water harvesting, preventing soil erosion along the banks and ensuring income and
livelihood opportunities for the local communities. Hence, statement 3 is correct.

https://odishasuntimes.com/odisha-cm-launches-green-mahanadi-mission-for-greener-river-basin/

9 With reference to the Cartagena Biosafety Protocol (CBP), consider the following statements:
1. It is an additional agreement to the convention on biological diversity (CBD).
2. It governs on the access to genetic resources and the fair and equitable sharing of benefits arising
from their utilization to the Convention of Biological Diversity (CBD).
3. India is a party to the protocol.

Which of the statements given above is/are correct?


A. 1 only
B. 2 and 3 only
C. 1 and 3 only
D. 1, 2 and 3

Correct Answer : C

Answer Justification :

The Cartagena Protocol on Biosafety to the Convention on Biological Diversity is an


international agreement which aims to ensure the safe handling, transport and use of
living modified organisms (LMOs) resulting from modern biotechnology that may have
adverse effects on biological diversity, taking also into account risks to human health. It was
adopted on 29 January 2000 as a supplementary agreement to the Convention on Biological
Diversity and entered into force on 11 September 2003.

Hence, statement 1 is correct and statement 2 is incorrect.

Totally, there are 171 parties to the protocol including India. Hence, statement 3 is
correct.

https://bch.cbd.int/protocol/

10 Consider the following pairs:


Interspecific interactions Example
1. Parasitism Koel laying its eggs in the nest of crow.
2. Amensalism Sea anemone and clown fish
3. Mutualism Fig tree and the pollinator species of wasp.

6
Total Marks : 200
Online Prelims TEST - 12 (SUBJECT WISE)
( InsightsIAS Mock Test Series for UPSC Preliminary Exam 2020 )

Which of the pairs given is/are correctly matched?


A. 1 and 2 only
B. 2 and 3 only
C. 1 and 3 only
D. 1, 2 and 3

Correct Answer : C

Answer Justification :

Brood parasitism in birds is a fascinating example of parasitism in which the parasitic bird
lays its eggs in the nest of its host and lets the host incubate them. During the course of
evolution, the eggs of the parasitic bird have evolved to resemble the host’s egg in size and colour
to reduce the chances of the host bird detecting the foreign eggs and ejecting them from the nest.
Hence, pair 1 is correctly matched.

Commensalism is a long-term biological interaction in which members of one species gain


benefits while those of the other species neither benefit nor are harmed. Example: Sea
anemone and clown fish. Hence, pair 2 is incorrectly matched.

Mutualism describes the ecological interaction between two or more species where each
species has a net benefit. Mutualism is thought to be the most common type of ecological
interaction, and it is often dominant in most communities worldwide. Example: Fig tree and
the pollinator species of wasp. Hence, pair 3 is correctly matched.

11 Consider the following statements:


1. In a particular climatic condition, decomposition rate is quicker if detritus is rich in lignin and
chitin.
2. Warm and moist environment favour decomposition whereas low temperature and anaerobiosis
inhibit decomposition resulting in building up of organic materials.

Which of the statements given above is/are correct?


A. 1 only
B. 2 only
C. Both 1 and 2
D. Neither 1 nor 2

Correct Answer : B

Answer Justification :

Decomposition is largely an oxygen-requiring process. The rate of decomposition is


controlled by chemical composition of detritus and climatic factors.

In a particular climatic condition, decomposition rate is slower if detritus is rich in lignin


and chitin, and quicker, if detritus is rich in nitrogen and water-soluble substances like

7
Total Marks : 200
Online Prelims TEST - 12 (SUBJECT WISE)
( InsightsIAS Mock Test Series for UPSC Preliminary Exam 2020 )

sugars. Hence, statement 1 is incorrect.

Temperature and soil moisture are the most important climatic factors that regulate decomposition
through their effects on the activities of soil microbes.

Warm and moist environment favour decomposition whereas low temperature and
anaerobiosis inhibit decomposition resulting in build up of organic materials. Hence,
statement 2 is correct.

12 “Colony Collapse Disorder” phenomenon is associated with which of the following?

A. Collapse entire colonies during urban floods


B. Coral bleaching
C. Death of bees
D. Collapse of islands due to climate change.

Correct Answer : C

Answer Justification :

Colony Collapse Disorder is the phenomenon that occurs when the majority of worker bees
in a colony disappear and leave behind a queen, plenty of food and a few nurse bees to care for
the remaining immature bees and the queen.

But hives cannot sustain themselves without worker bees and would eventually die.

Hence, option (c) is correct.

13 Consider the following statements:


1. In an aquatic ecosystem, detritus food chain is the major conduit for energy flow.
2. In a terrestrial ecosystem, a much larger fraction of energy flows through the grazing food chain
than through the detritus food chain.

Which of the statements given above is/are correct?


A. 1 only
B. 2 only
C. Both 1 and 2
D. Neither 1 nor 2

Correct Answer : D

Answer Justification :

In an aquatic ecosystem, Grazing food chain is the major conduit for energy flow. Hence,
statement 1 is incorrect.

8
Total Marks : 200
Online Prelims TEST - 12 (SUBJECT WISE)
( InsightsIAS Mock Test Series for UPSC Preliminary Exam 2020 )

As against this, in a terrestrial ecosystem, a much larger fraction of energy flows through the
detritus food chain than through the grazing food chain. Hence, statement 2 is incorrect.

Detritus food chain may be connected with the grazing food chain at some levels: some of the
organisms of DFC are prey to the GFC animals, and in a natural ecosystem, some animals like
cockroaches, crows, etc., are omnivores. These natural interconnection of food chains make it a
food web.

14 Consider the following statements:


1. The standing crop is measured as the mass of living organisms (biomass) or the number in a unit
area.
2. Measurement of biomass in terms of dry weight is more accurate than measuring in terms of wet
weight.

Which of the statements given above is/are correct?


A. 1 only
B. 2 only
C. Both 1 and 2
D. Neither 1 nor 2

Correct Answer : C

Answer Justification :

Each trophic level has a certain mass of living material at a particular


time called as the standing crop. The standing crop is measured as the
mass of living organisms (biomass) or the number in a unit area. Hence, statement 1 is
correct.

The biomass of a species is expressed in terms of fresh or dry weight.


Measurement of biomass in terms of dry weight is more accurate. Hence, statement 2 is
correct.

15 Which of the following ecological pyramids is/are always represented as upright pyramid?

A. Pyramid of Numbers
B. Pyramid of Energy flow
C. Pyramid of Biomass
D. None

Correct Answer : B

Answer Justification :

In the pyramid of energy flow, there is always loss of energy during the flow from one lower trophic

9
Total Marks : 200
Online Prelims TEST - 12 (SUBJECT WISE)
( InsightsIAS Mock Test Series for UPSC Preliminary Exam 2020 )

level to higher trophic levels. Hence they will always be upright.

16 Consider the following statements:


1. The species that invade a bare area are called keystone species.
2. Hydrarch succession takes place in wetter areas.
3. Xerarch succession takes place in dry areas.

Which of the statements given above is/are correct?


A. 1 and 2 only
B. 2 and 3 only
C. 1 and 3 only
D. 1, 2 and 3

Correct Answer : B

Answer Justification :

Ecological Succession is the gradual process by which ecosystems change and


develop over time

Based on the nature of the habitat – whether it is water (or very wet areas)
or it is on very dry areas – succession of plants is called hydrach or xerarch,
respectively.

Hydrarch succession takes place in wetter areas and the successional series progress from
hydric to the mesic conditions. Hence, statement 2 is correct.

As against this, xerarch succession takes place in dry areas and the series progress from
xeric to mesic conditions. Hence, both hydrarch and xerach
successions lead to medium water conditions (mesic) – neither too dry
(xeric) nor too wet (hydric). Hence, statement 3 is correct.

The species that invade a bare area are called pioneer species. In primary succession on rocks
these are usually lichens which are able to secrete acids to dissolve rock, helping in weathering and
soil formation. Hence, statement 1 is incorrect.

A keystone species is a species that has a disproportionately large effect on its natural environment
relative to its abundance. Without keystone species, the ecosystem would be dramatically different
or cease to exist altogether. Some keystone species, such as the wolf, tiger, lion etc, are also apex
predators.

17 Consider the following countries:


1. India
2. Brazil
3. China
4. Russia

10
Total Marks : 200
Online Prelims TEST - 12 (SUBJECT WISE)
( InsightsIAS Mock Test Series for UPSC Preliminary Exam 2020 )

Which of the countries given above is/are identified as Mega diverse country?
A. 1 and 2 only
B. 3 and 4 only
C. 1, 2 and 3 only
D. 1, 2, 3 and 4

Correct Answer : C

Answer Justification :

Megadiversity Countries is a term used to refer to the world’s top biodiversity-rich countries.

To qualify as a Megadiverse Country, a country must:

1. Have at least 5000 of the world’s plants as endemics

2. Have marine ecosystems within its borders.

The focus on endemism is in line with the IUCN’s “doctrine of ultimate responsibility”, which holds
that a country with the only populations of an endangered species has ultimate responsibility for
ensuring the survival of that particular species. Other secondary criteria have also been taken into
consideration, such as animal and invertebrate endemism, species diversity, higher-level diversity,
ecosystem diversity and presence of tropical rainforest ecosystems. Despite endemism being the
main criterion, thresholds for the criteria are flexible and countries have been considered
individually based on all criteria.

The identified Megadiverse Countries are: United States of America, Mexico, Colombia, Ecuador,
Peru, Venezuela, Brazil, Democratic Republic of Congo, South Africa, Madagascar, India, Malaysia,
Indonesia, Philippines, Papua New Guinea, China, and Australia.

Hence, option (c) is correct.

https://www.biodiversitya-z.org/content/megadiverse-countries

18 Which of the following processes can be categorized as part of ‘In-situ bioremediation’?


1. Biosparging
2. Bioventing
3. Bioaugmentation

Select the correct answer using the code given below:


A. 1 and 2 only
B. 2 and 3 only
C. 1 and 3 only
D. 1, 2 and 3

Correct Answer : D

11
Total Marks : 200
Online Prelims TEST - 12 (SUBJECT WISE)
( InsightsIAS Mock Test Series for UPSC Preliminary Exam 2020 )

Answer Justification :

Bioremediation is a process used to treat contaminated media, including water, soil and
subsurface material, by altering environmental conditions to stimulate growth of
microorganisms and degrade the target pollutants.

In situ Bioremediation means treatment of contaminated soil in the location where it was
found. In situ Bioremediation techniques include - Bioventing, Biosparging,
Bioaugmentation.

Bioventing is a process of stimulating the natural in situ biodegradation of contaminants


in soil by providing air or oxygen to existing soil microorganisms. Bioventing uses low air
flow rates to provide only enough oxygen to sustain microbial activity in the vadose zone

Biosparging is an in-situ remediation technology that uses indigenous microorganisms to


biodegrade organic constituents in the saturated zone. In biosparging, air (or oxygen) and
nutrients (if needed) are injected into the saturated zone to increase the biological activity of the
indigenous microorganisms.

Bioaugmentation is the practice of adding cultured microorganisms into the subsurface for the
purpose of biodegrading specific soil and groundwater contaminants.

19 Which of the following protected areas is/are the biosphere reserves in India?
1. Agasthyamalai
2. Pachmarhi
3. Nokrek
4. The Gulf of Mannar

Select the correct answer using the code given below:


A. 1 and 2 only
B. 2, 3 and 4 only
C. 1, 3 and 4 only
D. 1, 2, 3 and 4

Correct Answer : D

Answer Justification :

Biosphere reserves are sites established by countries and recognized under UNESCO's Man and the
Biosphere (MAB) Programme to promote sustainable development based on local community efforts
and sound science. The programme of Biosphere Reserve was initiated by UNESCO in 1971. The
purpose of the formation of the biosphere reserve is to conserve in situ all forms of life, along with
its support system, in its totality, so that it could serve as a referral system for monitoring and
evaluating changes in natural ecosystems.

12
Total Marks : 200
Online Prelims TEST - 12 (SUBJECT WISE)
( InsightsIAS Mock Test Series for UPSC Preliminary Exam 2020 )

Hence, option (d) is correct.

13
Total Marks : 200
Online Prelims TEST - 12 (SUBJECT WISE)
( InsightsIAS Mock Test Series for UPSC Preliminary Exam 2020 )

http://wiienvis.nic.in/Database/br_8225.aspx

20 With reference to the Biodiversity Hotspots, consider the following statements


1. One of the criteria to qualify as Biodiversity Hotspots is, it must contain at least 15,000 species of
endemic vascular plants.
2. Western ghats falls under a biological hotspot.
3. Norman Myers introduced concept of Biodiversity hotspots.

Which of the statements given above is/are correct?


A. 1 and 2 only
B. 2 and 3 only
C. 1 and 3 only
D. 1, 2 and 3

Correct Answer : B

Answer Justification :

The concept of a biodiversity hotspot was put forward by Norman Myers in 1988. Hence,
statement 3 is correct.

The criteria which are necessary for a region to be categorized as a


biodiversity hotspot include:
  Species endemism – it must contain at least 1500 species of vascular plants, which is more
than 0.5 percent of the world’s total plant population, as endemic species. Hence,
statement 1 is incorrect.
  Degree of threat – it must have lost at least 70 percent of its original habitat
due to human interventions.

There are 3 such hotspots in India. They include:

The Eastern Himalayas


Indo-Burma
The Western Ghats and Sri Lanka

Hence, statement 2 is correct.

https://www.cepf.net/our-work/biodiversity-hotspots/hotspots-defined

21 Consider the following species:


1. Asiatic Lion
2. Lion tailed macaque
3. Sangai Deer

Which of the species given above is/are endemic to India?


A. 2 and 3 only

14
Total Marks : 200
Online Prelims TEST - 12 (SUBJECT WISE)
( InsightsIAS Mock Test Series for UPSC Preliminary Exam 2020 )

B. 3 only
C. 1 and 2 only
D. 1, 2 and 3

Correct Answer : D

Answer Justification :

All the above given species are endemic to India.

Asiatic Lion also known as the Indian lions are listed as endangered and only found in and
around Gir Forest National Park of Gujarat. Gujarat Lion is one of five big cats inhabit India
others being the Bengal Tiger and Indian Leopards.

Lion Tailed Macaque is India’s endangered primate species and also ranks among the rarest and
most threatened monkey, endemic to the Western Ghats of South India.

Sangai also called as brow antlered deer is endemic and endangered species of deer, found
only in Keibul Lamjao National Park of Manipur. The Keibul Lamjao park is a marshy wetland
located at the south parts of the Loktak Lake.

Hence, option (d) is correct.

https://www.vedantu.com/biology/endemic-species

22 Consider the following statements:


1. The Conference of the Parties is the governing body of the Convention on Biodiversity (CBD).
2. Global Biodiversity Outlook (GBO) is a periodic report on biological diversity released by the
Convention on Biodiversity (CBD).
3. Agenda 21 is a legally binding action plan of the United Nations with regard to sustainable
development.

Which of the statements given above is/are correct?


A. 1 and 2 only
B. 2 and 3 only
C. 1 and 3 only
D. 1, 2 and 3

Correct Answer : A

Answer Justification :

The Convention on Biodiversity (CBD) was opened for signature on 5 June 1992 at the United
Nations Conference on Environment and Development (the Rio "Earth Summit").

The Convention on Biological Diversity was inspired by the world community's growing commitment

15
Total Marks : 200
Online Prelims TEST - 12 (SUBJECT WISE)
( InsightsIAS Mock Test Series for UPSC Preliminary Exam 2020 )

to sustainable development. It represents a dramatic step forward in the conservation of biological


diversity, the sustainable use of its components, and the fair and equitable sharing of benefits
arising from the use of genetic resources.

The Conference of the Parties is the governing body of the Convention, and advances
implementation of the Convention through the decisions it takes at its periodic meetings. Hence,
statement 1 is correct.

The second meeting of the Conference of the Parties called for the preparation of a
periodic report on biological diversity: the Global Biodiversity Outlook (GBO). It suggested
that the GBO should provide a summary of the status of biological diversity and an analysis of the
steps being taken by the global community to ensure that biodiversity is conserved and used
sustainably, and that benefits arising from the use of genetic resources are shared
equitably. Hence, statement 2 is correct.

Agenda 21 is a non-binding action plan of the United Nations with regard to sustainable
development. It is a product of the Earth Summit held in Rio de Janeiro, Brazil, in 1992. Hence,
statement 3 is incorrect.

https://www.cbd.int/cop/default.shtml

https://sustainabledevelopment.un.org/outcomedocuments/agenda21

23 Consider the following statements:


1. Electrostatic precipitator can remove most of the particulate matter present in the exhaust from a
thermal power plant.
2. A scrubber can remove gases like sulfur dioxide.
3. Motor vehicles equipped with catalytic converter should use leaded petrol because lead in the
petrol activates the catalyst.

Which of the statements given above is/are correct?


A. 1 and 2 only
B. 2 and 3 only
C. 1 and 3 only
D. 1, 2 and 3

Correct Answer : A

Answer Justification :

There are several ways of removing particulate matter; the most widely used of which is the
electrostatic precipitator, which can remove over 99 per cent particulate matter present in
the exhaust from a thermal power plant. It has electrode wires that are maintained at several
thousand volts, which produce a corona that releases electrons. These electrons attach to dust
particles giving them a net negative charge. The collecting plates are grounded and attract the
charged dust particles.
The velocity of air between the plates must be low enough to allow the dust to fall. Hence,

16
Total Marks : 200
Online Prelims TEST - 12 (SUBJECT WISE)
( InsightsIAS Mock Test Series for UPSC Preliminary Exam 2020 )

statement 1 is correct.

A scrubber can remove gases like sulphur dioxide. In a scrubber, the exhaust is passed through
a spray of water or lime. Hence, statement 2 is correct.

Catalytic converters, having expensive metals namely platinum-palladium and rhodium as the
catalysts, are fitted into automobiles for reducing emission of poisonous gases. As the
exhaust passes through the catalytic converter, unburnt hydrocarbons are converted into carbon
dioxide and water, and carbon monoxide and nitric oxide are changed to carbon dioxide and
nitrogen gas, respectively. Motor vehicles equipped with catalytic converter should use
unleaded petrol because lead in the petrol inactivates the catalyst. Hence, statement 3 is
incorrect.

24 With reference to the Compressed natural gas (CNG), consider the following statements:
1. CNG is comprised mostly of methane gas.
2. CNG engines run more quietly due to the higher octane rating of CNG over gasoline.

Which of the statements given above is/are correct?


A. 1 only
B. 2 only
C. Both 1 and 2
D. Neither 1 nor 2

Correct Answer : C

Answer Justification :

Compressed natural gas, or CNG, is natural gas under pressure which remains clear,
odourless, and non-corrosive – and can be used as a cheaper, greener, and more efficient
alternative to the traditional petrol and diesel fuels for vehicles.

CNG is comprised mostly of methane gas which, like gasoline, produces engine power when
mixed with air and fed into your engine's combustion chamber. Hence, statement 1 is correct.

CNG engines run more quietly due to the higher octane rating of CNG over gasoline and
they produce less exhaust emissions. Harmful emissions such as carbon monoxide (CO), carbon
dioxide (CO2 and nitrous oxide (N2O) can be reduced by as much as 95% when compared to
gasoline powered vehicles. Hence, statement 2 is correct.

https://www.cng.co.tt/what-is-cng/

25 With reference to Bharat VI norms (BS VI), consider the following statements:
1. The BS VI compliant vehicles will only release 50 ppm sulfur.
2. The BS VI norms are equivalent to Euro-VI norms currently in place across countries in Europe.
3. For the light duty diesel vehicles, the BS VI norms cut down the oxides of nitrogen emission limits

17
Total Marks : 200
Online Prelims TEST - 12 (SUBJECT WISE)
( InsightsIAS Mock Test Series for UPSC Preliminary Exam 2020 )

Which of the statements given above is/are correct?


A. 1 and 2 only
B. 2 and 3 only
C. 3 only
D. 1, 2 and 3

Correct Answer : B

Answer Justification :

The Supreme Court of India has ruled that no Bharat Stage IV vehicle shall be sold across the
country with effect from April 1, 2020. Instead, the Bharat Stage VI (or BS-VI) emission norm would
come into force from April 1, 2020 across the country.

The major difference between the existing BS-IV and forthcoming BS-VI norms is the presence of
sulphur in the fuel. While the BS-IV fuels contain 50 parts per million (ppm) sulphur, the BS-VI
grade fuel only has 10 ppm sulphur content. Hence, statement 1 is incorrect.

The harmful NOx (nitrogen oxides) from diesel cars can be brought down by nearly 70%. In
the petrol cars, they can be reduced by 25%. However, when we talk air pollution, particulate
matter like PM 2.5 and PM 10 are the most harmful components and the BS VI will bring the cancer
causing particulate matter in diesel cars by a phenomenal 80%. Hence, statement 3 is correct.

The BS VI norms is equivalent to Euro-VI norms currently in place across countries in Europe.
Hence, statement 2 is correct.
https://www.businessinsider.in/what-is-bs-vi-norms/articleshow/69286150.cms?utm_source=content
ofinterest&utm_medium=text&utm_campaign=cppst

26 With reference to the Biomining , consider the following statements:


1. It is the process of using microorganisms (microbes) to extract metals of economic interest
from rock ores or mine waste.
2. Most current biomining operations target valuable metals like copper, uranium, nickel, and gold
etc.

Which of the statements given above is/are correct?


A. 1 only
B. 2 only
C. Both 1 and 2
D. Neither 1 nor 2

Correct Answer : C

Answer Justification :

Biomining is the process of using microorganisms (microbes) to extract metals of


economic interest from rock ores or mine waste. Biomining techniques may also be used to

18
Total Marks : 200
Online Prelims TEST - 12 (SUBJECT WISE)
( InsightsIAS Mock Test Series for UPSC Preliminary Exam 2020 )

clean up sites that have been polluted with metals. Hence, statement 1 is correct.

Most current biomining operations target valuable metals like copper, uranium, nickel,
and gold that are commonly found in sulfidic (sulfur-bearing) minerals. Microbes are especially
good at oxidizing sulfidic minerals, converting metals like iron and copper into forms that can
dissolve more easily. Other metals, like gold, are not directly dissolved by this microbial process,
but are made more accessible to traditional mining techniques because the minerals surrounding
these metals are dissolved and removed by microbial processes. When the metal of interest is
directly dissolved, the biomining process is called “bioleaching,” and when the metal of interest is
made more accessible or “enriched” in the material left behind, it is called “biooxidation.” Both
processes involve microbial reactions that can happen anywhere the microbes, rocks, and necessary
nutrients, like oxygen, occur together. Hence, statement 2 is correct.

https://www.americangeosciences.org/critical-issues/faq/what-biomining

27 The index titled ' Environmental Democracy Index' sometimes seen in the news, released by

A. Green Peace
B. IUCN
C. World Bank
D. World Resource Institute

Correct Answer : D

Answer Justification :

The World Resources Institute is the Secretariat of the Access Initiative (TAI)- the world’s largest
civil society network dedicated to ensuring that people have the right and ability to influence
decisions that affect their communities and the natural resources on which they rely.

WRI launched the first ever Environmental Democracy Index (EDI) on May 20, 2015, the first
index to measure how well countries' national laws protect environmental democracy rights.

Hence, option (d) is correct.

https://www.wri.org/our-work/project/environmental-democracy-index

28 Which of the following is not included in India’s National Action Plan on Climate Change (NAPCC)?

A. National Mission for Enhanced Energy Efficiency


B. National Soil Mission
C. National Water Mission
D. National Mission for Sustainable Agriculture

Correct Answer : B

19
Total Marks : 200
Online Prelims TEST - 12 (SUBJECT WISE)
( InsightsIAS Mock Test Series for UPSC Preliminary Exam 2020 )

Answer Justification :

India’s National Action Plan on Climate Change (NAPCC) which covers eight major missions on
Solar, Enhanced Energy Efficiency, Sustainable Habitat, Water, Sustaining the Himalayan
Ecosystem, Green India, Sustainable Agriculture and Strategic Knowledge on Climate
Change.

Hence, option (b) is correct.

29 Which of the following amendment added new emission reduction targets for Second Commitment
Period (2012-2020) of Kyoto Protocol?

A. Doha Amendment
B. Durban Amendment
C. Paris Agreemenet
D. Kigali Amendment.

Correct Answer : A

Answer Justification :

The Kyoto Protocol was adopted in 1997 to fight global warming by reducing
GHGs emission and came into effect in 2005. The 1st commitment period under the Kyoto
Protocol was from 2008-2012. The 2nd commitment period for the period 2013- 2020 was
adopted in 2012 by the Doha Amendment of the Kyoto Protocol.

Hence, option (a) is correct.

30 ‘Genetic Engineering Appraisal Committee’ (GEAC) is under the aegis of

A. The Environment (Protection) Act, 1986


B. The Wildlife Protection Act, 1972
C. Biological Diversity Act, 2002
D. The Forest (Conservation) Act, 1927

Correct Answer : A

Answer Justification :

Under the The Environment (Protection) Act, 1986 "Rules for Manufactute, Use, Import, Export and
20
Total Marks : 200
Online Prelims TEST - 12 (SUBJECT WISE)
( InsightsIAS Mock Test Series for UPSC Preliminary Exam 2020 )

Storage of Hazardous Microorganisms/Genetically Engineered Organisms or Cells 1989", GEAC is


responsible for granting permits to conduct experimental and large-scale open field trials and also
grant approval for commercial release of biotech crops.

Hence, option (a) is correct.

31 Consider the following statements:


1. High concentrations of DDT result in the increase of thickness of eggshell.
2. Eutrophication results in depletion of dissolved oxygen in water.
3. Biomagnification refers to increase in concentration of the toxicant at successive trophic levels.

Which of the statements given above are correct?


A. 1 and 2 only
B. 2 and 3 only
C. 3 only
D. 1, 2 and 3

Correct Answer : B

Answer Justification :

Biomagnification refers to increase in concentration of the toxicant at successive trophic


levels. This happens because a toxic substance accumulated by an organism cannot be metabolised
or excreted, and is thus passed on to the next higher trophic level. This phenomenon is well-known
for mercury and DDT. Hence, statement 3 is correct.

High concentrations of DDT disturb calcium metabolism in birds, which causes


thinning of eggshell and their premature breaking, eventually causing decline in bird populations.
Hence, statement 1 is incorrect.

Eutrophication is the natural aging of a lake by biological enrichment of its water. The prime
contaminants are nitrates and phosphates, which act as plant nutrients. They over stimulate the
growth of algae, causing unsightly scum and unpleasant odors, and robbing the water of
dissolved oxygen vital to other aquatic life. At the same time, other pollutants flowing into a
lake may poison whole populations of fish, whose decomposing remains further deplete the
water’s dissolved oxygen content. In such fashion, a lake can literally choke to death. Hence,
statement 2 is correct.

32 Which of the following serves as International Corridor for Asian Elephant Migration between India
and Bhutan?

A. The Tale Life Sanctuary


B. Buxa Tiger Reserve
C. Valmiki Tiger Reserve
D. Namdapha Tiger Reserve

21
Total Marks : 200
Online Prelims TEST - 12 (SUBJECT WISE)
( InsightsIAS Mock Test Series for UPSC Preliminary Exam 2020 )

Correct Answer : B

Answer Justification :

The Buxa Tiger Reserve is a 760-square-kilometre (290 sq mi) tiger reserve located inside the Buxa
National Park in West Bengal, India, in the Buxa Hills of the southern hilly area of Bhutan. Animals
found in the park include, the Tiger, Civet, Elephant, Gaur (Indian bison), Indian Boar and Red
jungle fowl.

Buxa Tiger Reserve (BTR) lies in Alipurduar district of West Bengal. Its northern boundary
runs along the international border with Bhutan. The Sinchula hill range lies all along the
northern side of BTR and the eastern boundary touches that of the Assam State. National
Highway No.31 C roughly runs along its southern boundary. It is the eastern most extension of
extreme bio-diverse North-East India & represents highly endemic Indo-Malayan region. The fragile
“Terai Eco-System” constitutes a part of this Reserve. The Phipsu Wildlife Sanctuary of Bhutan is
contiguous to North of BTR. Manas National Park lies on east of BTR. BTR, thus, serves as
international corridor for Asian elephant migration between India and Bhutan.

Hence, option (c) is correct.

33 Consider the following chemical elements:


1. Barium
2. Cadmium
3. Lead
4. Mercury

Which of the chemicals given above is/are part of E-waste?


A. 1, 2 and 3 only
B. 2, 3 and 4 only
C. 3 and 4 only
D. 1, 2, 3 and 4

Correct Answer : D

Answer Justification :

Irreparable computers and other electronic goods are known as electronic wastes (e-wastes). E-
wastes are buried in landfills or incinerated. Over half of the e-wastes generated in the
developed world are exported to developing countries, mainly to China, India and Pakistan,
where metals like copper, iron, silicon, nickel and gold are recovered.

Barium is a soft silvery-white metal that is used in computers in the front panel of a CRT to protect
users from radiation.

Cadmium occurs in SMD chip resistors, infrared detectors, and semiconductor chips. Some older
cathode ray tubes contain cadmium.

22
Total Marks : 200
Online Prelims TEST - 12 (SUBJECT WISE)
( InsightsIAS Mock Test Series for UPSC Preliminary Exam 2020 )

Lead is used in glass panels and gaskets in computer monitors, solder in printed circuit boards and
other components

Mercury is used in thermostats, sensors, relays, switches, medical equipment,


lamps, mobile phones, and batteries.

34 Which of the following organization/institution publishes the ‘Living Planet Report’?

A. UNFCCC
B. UNEP
C. World Wide Fund for Nature (WWF)
D. World Bank

Correct Answer : C

Answer Justification :

The Living Planet Report, WWF’s flagship publication released every two years, is a
comprehensive study of trends in global biodiversity and the health of the planet.

Hence, option (c) is correct.

35 With reference to the Climate and Clean Air Coalition, consider the following statements:
1. It is a voluntary partnership of governments, intergovernmental organizations, businesses, scientific
institutions and civil society organizations
2. It aims to reduce long-lived climate pollutants .

Which of the statements given above is/are correct?


A. 1 only
B. 2 only
C. Both 1 and 2
D. Neither 1 nor 2

Correct Answer : A

Answer Justification :

The Climate and Clean Air Coalition is a voluntary partnership of governments,


intergovernmental organizations, businesses, scientific institutions and civil society
organizations committed to improving air quality and protecting the climate through actions to
reduce short-lived climate pollutants. Hence, statement 1 is correct and statement 2 is
incorrect.

36 With reference to the ‘Central Zoo Authority (CZA)’, consider the following statements:

23
Total Marks : 200
Online Prelims TEST - 12 (SUBJECT WISE)
( InsightsIAS Mock Test Series for UPSC Preliminary Exam 2020 )

1. It was established under the Wildlife Protection Act, 1972.


2. It is a statutory body under the Ministry of Environment, Forest and Climate Change.

Which of the statements given above is/are correct?


A. 1 only
B. 2 only
C. Both 1 and 2
D. Neither 1 nor 2

Correct Answer : C

Answer Justification :

Central Zoo Authority was established as a statutory body under the Ministry of Environment
& Forests by the Government of India in the year 1992. Hence, statement 2 is correct.

The Authority consists of a Chairman, ten members and a Member Secretary. Section 38 A to 38 J
was added to the Wildlife (Protection) Act, 1972 for establishment of the Central Zoo Authority
in India. Hence, statement 1 is correct.

http://cza.nic.in/

37 Consider the following pairs:


Pollutant Associated Disease
1. Nitrates Minimata
2. Mercury Black foot disesase
3. Cadmium Blue Baby Syndrome

Which of the above pairs is/are correctly matched?


A. 1 and 2 only
B. 1, 2 and 3
C. 3 only
D. None

Correct Answer : D

Answer Justification :

Minamata Disease is a crippling deformity caused by methyl mercury poisoning that was first
described in the inhabitants of Minamata Bay, Japan and resulted from their eating fish
contaminated with mercury industrial waste. Hence, pair 1 is incorrectly matched.

Black foot Disease is caused by leaching of arsenic from soil & rocks to ground water used for
drinking purposes. Hence, pair 2 is incorrectly matched.

24
Total Marks : 200
Online Prelims TEST - 12 (SUBJECT WISE)
( InsightsIAS Mock Test Series for UPSC Preliminary Exam 2020 )

Excess nitrates in drinking water reacts with hemoglobin to form non-functional methaemoglobin
which impairs oxygen transport. It is called Bluebaby syndrome. Hence, pair 3 is incorrectly
matched.

Water contaminated with cadmium can cause itai itai desease

38 Which of the following states is the first state to adopt Fly Ash Utilization Policy in India?

A. Maharashtra
B. Karnataka
C. Punjab
D. Tamil Nadu

Correct Answer : A

Answer Justification :

Maharashtra has become the first state to adopt Fly Ash Utilization Policy, paving way for
prosperity by generating "wealth from waste", and environment protection.

https://www.business-standard.com/article/pti-stories/maha-becomes-first-state-to-adopt-fly-ash-utili
sation-policy-116111500919_1.html

39 Consider the following statements:


1. India is the founding member of the Global Tiger Forum (GTF).
2. Pakke Tiger reserve is located in the state of Arunachal Pradesh.
3. National Tiger Conservation Authority (NTCA) statutory body constituted under the Wildlife
(Protection) Act, 1972 headed by the Prime Minister.

Which of the statements given above is/are correct?


A. 1 and 2 only
B. 2 and 3 only
C. 3 only
D. 1, 2 and 3

Correct Answer : A

Answer Justification :

The Global Tiger Forum (GTF) is the only inter- governmental international body established
with members from willing countries to embark on a global campaign to protect the Tiger.

Utilizing co-operative policies, common approaches, technical expertise, scientific modules and

25
Total Marks : 200
Online Prelims TEST - 12 (SUBJECT WISE)
( InsightsIAS Mock Test Series for UPSC Preliminary Exam 2020 )

other appropriate programmes and controls the GTF is focused on saving the remaining 5 sub-
species of Tigers distributed over 13 Tiger Range countries of the world.

The GTF was formed in 1993 on recommendations from an international symposium on Tiger
Conservation at New Delhi, India. The first meeting of the Tiger Range countries to setup the
forum was held in 1994, in which India was elected to the Chair and was asked to form an
interim secretariat. Hence, statement 1 is correct.

Pakke Tiger Reserve, also known as Pakhui Tiger Reserve, is a Project Tiger reserve in the East
Kameng district of Arunachal Pradesh in northeastern India. Hence, statement 2 is correct.

National Tiger Conservation Authority (NTCA) statutory body constituted under the Wildlife
(Protection) Act, 1972 headed by the Ministry of Environment, Forest and Climate Change.
Hence, statement 3 is incorrect.

http://globaltigerforum.org/about-gtf/

https://projecttiger.nic.in/

40 Which one of the following is an example for ex-situ method of conservation of flora and fauna?

A. Wildlife sanctuary
B. Botanical garden
C. Sacred Groves
D. Tiger Reserve

Correct Answer : B

Answer Justification :

Ex situ conservation is the technique of conservation of all levels of biological diversity outside
their natural habitats through different techniques like zoo, captive breeding, aquarium,
botanical garden, and gene bank.

Hence, option (b) is correct.

41 With reference to the Critical Ecosystem Partnership Fund (CEPF), consider the following
statements:
1. The fund is a joint program of Conservation International, the European Union, the Global
Environment Facility, the Government of India and the United Nations Environment Programme
(UNEP).
2. It aims to protect world’s biodiversity hotspots in developing and transitional economies by
empowering civil societies and individuals.

Which of the statements given above is/are correct?


A. 1 only

26
Total Marks : 200
Online Prelims TEST - 12 (SUBJECT WISE)
( InsightsIAS Mock Test Series for UPSC Preliminary Exam 2020 )

B. 2 only
C. Both 1 and 2
D. Neither 1 nor 2

Correct Answer : B

Answer Justification :

Biodiversity—the rich array of life on Earth—is fundamental to human survival, but under
tremendous and growing threat. The Critical Ecosystem Partnership Fund (CEPF) was
founded in 2000 to address this challenge by empowering civil society in developing
countries and transitional economies to protect the world’s biodiversity hotspots, which are
some of Earth's most biologically rich yet threatened terrestrial ecosystems. Hence, statement 2 is
correct.

The fund is a joint program of l’Agence Française de Développement, Conservation


International, the European Union, the Global Environment Facility, the Government of
Japan and the World Bank. Hence, statement 1 is incorrect.

https://www.cepf.net/about

42 Which of the following states recently declared Buddha Peacock or Buddha Mayoori as its State
butterfly?

A. Telangana
B. Maharashtra
C. Kerala
D. Andhra Pradesh

Correct Answer : C

Answer Justification :

Buddha Peacock or Buddha Mayoori, which was recently declared as the State butterfly
of Kerala.

Hence, option (c) is correct.

https://www.thehindu.com/news/national/kerala/a-forest-filled-with-butterflies/article25533169.ece

43 Which of the following organizations launched the ‘Global wildlife Program (GWP)’?

A. The Convention on International Trade in Endangered Species of Wild Fauna and Flora
(CITES)
B. International Union for Conservation of Nature (IUCN)

27
Total Marks : 200
Online Prelims TEST - 12 (SUBJECT WISE)
( InsightsIAS Mock Test Series for UPSC Preliminary Exam 2020 )

C. World Bank
D. United Nations Environment Programme (UNEP)

Correct Answer : C

Answer Justification :

The Global wildlife Program (GWP) is a World-Bank led global partnership that promotes
wildlife conservation and sustainable development by combatting illicit trafficking in wildlife.

https://www.worldbank.org/en/programs/global-wildlife-program

44 With reference to the Vermin species, consider the following statements:


1. They are defined under schedule VI of the Wildlife Protection Act, 1972.
2. As per the act, States can send a list of wild animals to the Centre requesting it to declare them
vermin for selective slaughter.
3. The Central Government may by notification, declare any wild animal to be vermin for any area for
a given period of time.

Which of the statements given above is/are correct?


A. 1 and 2 only
B. 2 only
C. 1 and 3 only
D. None

Correct Answer : B

Answer Justification :

Vermin means wild mammals and birds which are harmful to crops, farm animals or which carry
disease

As per Section 62 of the Wildlife Protection Act, 1972, States can send a list of wild animals to
the Centre requesting it to declare them vermin for selective slaughter. Hence, statement
2 is correct.

The Central Government may by notification, declare any wild animal other than those
specified in Schedule I and part 11 of Schedule H of the law to be vermin for any area for a
given period of time. Hence, statement 3 is incorrect.

As long as the notification is in force such wild animal shall be included in Schedule V of the
law, depriving them of any protection under that law. Hence, statement 1 is incorrect.

https://www.thehindu.com/specials/Vermin-or-victim/article14428071.ece

28
Total Marks : 200
Online Prelims TEST - 12 (SUBJECT WISE)
( InsightsIAS Mock Test Series for UPSC Preliminary Exam 2020 )

45 With reference to the International Union for Conservation of Nature (IUCN), consider the following
statements:
1. It is composed of both government and civil society organizations.
2. It has observer and consultative status at the United Nations.
3. The World Conservation Congress (Members’ Assembly) is the principal governing body of IUCN.

Which of the statements given above is/are correct?


A. 3 only
B. 2 and 3 only
C. 1 and 2 only
D. 1, 2 and 3

Correct Answer : C

Answer Justification :

he International Union for Conservation of Nature (IUCN) is a membership Union uniquely


composed of both government and civil society organizations. It provides public, private and
non-governmental organizations with the knowledge and tools that enable human progress,
economic development and nature conservation to take place together. Hence, statement 1 is
correct.

With its official United Nations Observer Status, IUCN ensures that nature conservation has a
voice at the highest level of international governance. Hence, statement 2 is correct.

The IUCN Council is the principal governing body of IUCN, International Union for
Conservation of Nature, in between sessions of the World Conservation Congress - the general
assembly of the Union's members.

The Members’ Assembly is IUCN’s highest decision-making body. Hence, statement 3 is


incorrect.

https://www.iucn.org/about

46 Consider the following statements:


1. South Asia Wildlife Enforcement Network (SAWEN) is an inter-governmental wildlife law
enforcement support body of all SAARC nations.
2. The International Consortium on Combating Wildlife Crime (ICCWC) is an alliance of the
Convention on International Trade in Endangered Species of Wild Fauna and Flora (CITES)
Secretariat, INTERPOL, the United Nations Office on Drugs and Crime, the World Bank and the
World Customs Organization.

Which of the statements given above is/are correct?


A. 1 only
B. 2 only
C. Both 1 and 2
D. Neither 1 nor 2

29
Total Marks : 200
Online Prelims TEST - 12 (SUBJECT WISE)
( InsightsIAS Mock Test Series for UPSC Preliminary Exam 2020 )

Correct Answer : C

Answer Justification :

South Asia Wildlife Enforcement Network (SAWEN) is an inter-governmental wildlife law


enforcement support body of South Asian countries namely - Afghanistan, Bangladesh,
Bhutan, India, Maldives, Nepal, Pakistan and Sri Lanka. SAWEN was officially launched in
January, 2011 in Paro Bhutan. Hence, statement 1 is correct.

ICCWC is the collaborative effort of five inter-governmental organizations working to bring


coordinated support to the national wildlife law enforcement agencies and to the sub-regional and
regional networks that, on a daily basis, act in defense of natural resources. The ICCWC partners
are the Convention on International Trade in Endangered Species of Wild Fauna and Flora
(CITES) Secretariat, INTERPOL, the United Nations Office on Drugs and Crime, the World
Bank and the World Customs Organization. This powerful alliance was formally established on
23 November 2010 in St. Petersburg, Russia during the International Tiger Forum.

Hence, statement 2 is correct.

https://www.sawen.org/pages/details/about

https://www.cites.org/eng/prog/iccwc.php

47 Consider the following statements:


1. National Board for wildlife is a statutory organization constituted under the Forest Conservation
Act, 1980.
2. The Animal Welfare Board of India is a statutory body established under the Wildlife Protection Act,
1972.
3. The Wildlife Institute of India is an autonomous institution under the Ministry of Environment
Forest and Climate change, Government of India.

Which of the statements given above is/are correct?


A. 3 only
B. 2 and 3 only
C. 1 and 2 only
D. 1, 2 and 3

Correct Answer : A

Answer Justification :

National Board for wildlife is a “Statutory Organization” constituted under the Wildlife Protection
Act, 1972. Hence, statement 1 is incorrect.

The Animal Welfare Board of India is a statutory advisory body on Animal Welfare Laws
and promotes animal welfare in the country. Established in 1962 under Section 4 of the

30
Total Marks : 200
Online Prelims TEST - 12 (SUBJECT WISE)
( InsightsIAS Mock Test Series for UPSC Preliminary Exam 2020 )

Prevention of Cruelty to Animals Act, 1960), the Animal Welfare Board of India was started
under the stewardship of Late Smt. Rukmini Devi Arundale, well known humanitarian.

The Board consists of 28 Members. The term of office of Members is for a period of 3 years. Hence,
statement 2 is incorrect.

The Wildlife Institute of India is an autonomous institution under the Ministry of


Environment Forest and Climate change, Government of India. established in May 1982. Hence,
statement 3 is correct.

http://www.awbi.in/about.html

https://www.insightsonindia.com/2019/02/14/national-board-for-wildlife-nbwl/

48 Consider the following sites:


1. Dal lake
2. Majuli
3. Ambaraguda
4. Nallur Tamarind Grove

Which of the sites given above is/are classified as Biodiversity Heritage Sites?
A. 1, 2 and 3 only
B. 2, 3 and 4 only
C. 1 and 3 only
D. 1, 2 , 3 and 4

Correct Answer : B

Answer Justification :

Biodiversity Heritage Sites” (BHS) are well defined areas that are unique, ecologically fragile
ecosystems - terrestrial, coastal and inland waters and, marine having rich biodiversity comprising
of any one or more of the following components: richness of wild as well as domesticated
species or intra-specific categories, high endemism, presence of rare and threatened
species, keystone species, species of evolutionary significance, wild ancestors of domestic/
cultivated species or their varieties, past pre-eminence of biological components represented
by fossil beds and having significant cultural, ethical or aesthetic values and are important for
the maintenance of cultural diversity, with or without a long history of human association with
them.

List of Biodiversity Heritage Sites

31
Total Marks : 200
Online Prelims TEST - 12 (SUBJECT WISE)
( InsightsIAS Mock Test Series for UPSC Preliminary Exam 2020 )

32
Total Marks : 200
Online Prelims TEST - 12 (SUBJECT WISE)
( InsightsIAS Mock Test Series for UPSC Preliminary Exam 2020 )

Dal Lake is not a Biodiversity Heritage Site.

Hence, option (b) is correct.

http://wiienvis.nic.in/Database/bhs_8650.aspx

49 With reference to the Dugong species, consider the following statements:


1. It is the only member of the Order Sirenia found in India.
2. It grazes on the sea grass meadows in coastal waters.

Which of the statements given above is/are correct?


A. 1 only
B. 2 and 3 only
C. 1 and 3 only
D. 1, 2 and 3

Correct Answer : D

33
Total Marks : 200
Online Prelims TEST - 12 (SUBJECT WISE)
( InsightsIAS Mock Test Series for UPSC Preliminary Exam 2020 )

Answer Justification :

Dugong (Dugong dugon) is the only herbivorous mammal that is strictly marine and the only
member of the Order Sirenia found in India. Hence, statement 1 is correct.

Dugongs are restricted to coastal shallow marine habitats and grazes on the sea grass
meadows in coastal waters and are therefore called as “Sea Cows”. Hence, statement 2 is
correct.

In India, it is one of the most seriously endangered species of large mammals. Dugongs are
vulnerable to anthropogenic pressures as they are solely dependent on sea grasses in coastal areas,
which now have been seriously damaged by mining, trawling etc. Dugongs have also been hunted
for their meat, oil, hides, bones and teeth.

It is listed in the schedule I of Wildlife Protection Act, 1972 and appendix I of CITES. It is classified
as Vulnerable in IUCN Red list. Hence, statement 3 is correct. Hence, statement 3 is correct

http://wiienvis.nic.in/Database/SRP_8555.aspx#4._Dugong__(click_on_name_to_go_top)

50 Angria Bank, which was seen in news recently, belongs to which of the following states?

A. Andhra Pradesh
B. Maharashtra
C. Gujarat
D. Karnataka

Correct Answer : B

Answer Justification :

Angria Bank is a bank, a shallow sunken atoll, on the continental shelf off the west coast of India.
It is located 105 kilometres west of Vijaydurg, Maharashtra. Hence, option (b) is correct.

51 Which among the following is the most recent to join the list of Ramsar Wetlands in India?

A. Sunderbans
B. Renuka Wetlands
C. Nalsarovar Bird Sanctuary
D. Point Calimere

Correct Answer : A

Answer Justification :

The Ramsar Convention is an international treaty for the conservation and sustainable utilization of

34
Total Marks : 200
Online Prelims TEST - 12 (SUBJECT WISE)
( InsightsIAS Mock Test Series for UPSC Preliminary Exam 2020 )

wetlands, recognizing the fundamental ecological functions of wetlands and their economic,
cultural, scientific, and recreational value.

Hence, option (a) is correct.

https://www.ramsar.org/news/indian-sundarbans-named-as-a-wetland-of-international-importance

http://wiienvis.nic.in/Database/ramsar_wetland_sites_8224.aspx

52 Consider the following statements:


1. Sundari trees are mangrove species which is classified as near threatened as IUCN red data book.
2. Mangroves for the Future initiative was developed by IUCN and UNEP.
3. Bhitarkanika is a second largest mangrove region in Indian subcontinent after Sunderbans region.

Which of the statements given above is/are correct?


A. 3 only
B. 2 and 3 only
C. 1 and 3 only
D. 1, 2 and 3

Correct Answer : A

Answer Justification :

35
Total Marks : 200
Online Prelims TEST - 12 (SUBJECT WISE)
( InsightsIAS Mock Test Series for UPSC Preliminary Exam 2020 )

Heritiera fomes is a species of mangrove tree in the family Malvaceae. Its common names include
sunder, sundri, jekanazo and pinlekanazo. It is the dominant mangrove tree species of the
Sundarbans of Bangladesh and India, and comprises about 70% of the trees in the area. It is
classified as endangered species. Hence, statement 1 is incorrect.

The devastation caused by the Indian Ocean tsunami of December 2004 laid bare the vital link
between coastal ecosystems and human livelihoods. It was United States President Bill Clinton’s
vision that rebuilding in tsunami-hit areas should improve natural infrastructure and strengthen
resilience against future natural disasters. In response to this vision, IUCN (International Union
for Conservation of Nature) and the United Nations Development Programme (UNDP)
developed Mangroves for the Future in 2006.

Mangroves for the Future (MFF) is a unique partner-led initiative to promote investment in coastal
ecosystem conservation for sustainable development. Co-chaired by IUCN and UNDP, MFF
provides a platform for collaboration among the many different agencies, sectors and countries
which are addressing challenges to coastal ecosystem and livelihood issues. Hence, statement 2 is
incorrect.

Bhitarkanika is a second largest mangrove region in Indian subcontinent after Sunderbans


region located in Odisha. It is also known for nesting site for Olive Ridley sea turtles. Hence,
statement 3 is correct.

https://www.iucn.org/regions/asia/our-work/regional-projects/mangroves-future-mff

53 Consider the following statements:


1. The National Compensatory Afforestation Fund falls under the Public Account of India.
2. As per India State of Forest Report, 2017, India’s forest cover has registered a marginal decrease of
about one per cent between 2015 and 2017.
3. The Champions of the Earth award is the highest environmental award of United Nations.

Which of the statements given above is/are correct?


A. 3 only
B. 2 and 3 only
C. 1 and 3 only
D. 1, 2 and 3

Correct Answer : C

Answer Justification :

The objective Compensatory Afforestation Fund Management and Planning Authority (CAMPA) is to
promote afforestation and regeneration activities as a way of compensating for forest land diverted
to non-forest uses

The National Compensatory Afforestation Fund under the Public account of India, and a State
Compensatory Afforestation Fund under the Public account of each state. Hence, statement 1
is correct.

36
Total Marks : 200
Online Prelims TEST - 12 (SUBJECT WISE)
( InsightsIAS Mock Test Series for UPSC Preliminary Exam 2020 )

As per India State of Forest Report, 2017, India’s forest cover has registered a marginal increase
of about one per cent between 2015 and 2017. Hence, statement 2 is incorrect.

Champions of the Earth, the UN's highest environmental honour, celebrates outstanding
figures from the public and private sectors and from civil society whose actions have had a
transformative positive impact on the environment. Hence, statement 3 is correct.

https://economictimes.indiatimes.com/news/et-explains/what-is-champions-of-the-earth-award-and-w
hy-has-narendra-modi-won-it/articleshow/66052143.cms

https://economictimes.indiatimes.com/news/politics-and-nation/transfer-rs-54000-crore-campa-funds
-to-centre-supreme-court/articleshow/67733209.cms

54 With reference to the Shola Forests, consider the following statements:


1. They are patches of stunted temperate montane forest.
2. They have spread over the states of Karnataka, Tamil Nadu and Kerala.
3. They have high concentration of lichen, mosses, ferns and orchids.

Which of the statements given above is/are correct?


A. 1 only
B. 2 and 3 only
C. 2 only
D. 1, 2 and 3

Correct Answer : B

Answer Justification :

The Shola vegetation are tropical montane forests found in the Western Ghats separated by
rolling grasslands in high altitudes. Hence, statement 1 is incorrect.

These patches of shola forest are found mainly in the valleys and are usually separated from one
another by undulating montane grassland.

Shola forests are found in the higher elevation hill regions of the Nilgiris, Anaimalai, Munnar, Palni
hills, Meghamalai, Agasthyamalai to the south and the Malnad and associated ranges in parts
of Wayanad, Coorg, Baba Budangiri and Kudremukh up the north, in the states
of Karnataka, Kerala and Tamil Nadu. Hence, statement 2 is correct.

They have high concentration of lichen, mosses, ferns and orchids. Hence, statement 3 is
correct.

https://www.downtoearth.org.in/news/environment/invasive-species-may-soon-wipe-out-shola-vegeta
tion-from-nilgiris-report-66813

55 Consider the following statements:


1. They are shrubs found in parts of the Western Ghats.
37
Total Marks : 200
Online Prelims TEST - 12 (SUBJECT WISE)
( InsightsIAS Mock Test Series for UPSC Preliminary Exam 2020 )

2. They blossom only once in 12 year.


3. The Paliyan tribal people living in Tamil Nadu used it as a reference to calculate their age.

The above given characteristics refer to which of the following flower?


A. Malabar Fingersop
B. Ceylon Green Champa
C. Neelakurinji
D. Nilgiri Champa

Correct Answer : C

Answer Justification :

Strobilanthes kunthiana, kurinji or neelakurinji, is a shrub that is found in the shola


forests of the Western Ghats in South India. Nilgiri Hills, which literally means the blue
mountains, got their name from the purplish blue flowers of Neelakurinji that blossoms only
once in 12 years.

The Paliyan tribal people living in Tamil Nadu used it as a reference to calculate their age.

Hence, option (c) is correct.

56 Consider the following statements:


1. India is a member state of the International Bamboo and Rattan Organization (INBAR).
2. Among the tiger reserves of India, Orang tiger reserve has highest tiger density.

Which of the statements given above is/are correct?


A. 1 only
B. 2 only
C. Both 1 and 2
D. Neither 1 nor 2

Correct Answer : C

Answer Justification :

The International Bamboo and Rattan Organisation (INBAR) is a multilateral development


organisation that promotes environmentally sustainable development using bamboo and rattan. It
has 45 Member States. In addition to its Secretariat headquarters in China, INBAR has Regional
Offices in Cameroon, Ecuador, Ethiopia, Ghana and India. Hence, statement 1 is correct.

Among the tiger reserves of India, Orang tiger reserve has highest tiger density.

Density is calculated based on the number of tigers per 100 sq. km. “If we calculate density of tigers
for 100 sq. km., it comes to 35.44. Thus, Orang has the highest density nationally. Hence,
statement 2 is correct.

38
Total Marks : 200
Online Prelims TEST - 12 (SUBJECT WISE)
( InsightsIAS Mock Test Series for UPSC Preliminary Exam 2020 )

https://www.inbar.int/about-inbar/#2

https://www.thehindu.com/news/national/other-states/tiny-orang-roars-on-tiger-density/article18718
989.ece

57 Consider the following statements:


1. Conservation reserves were first introduced in the Wildlife (Protection) Amendment Act of 2002.
2. Kerala state has the highest number of community reserves amongst all the states.

Which of the statements given above is/are correct?


A. 1 only
B. 2 only
C. Both 1 and 2
D. Neither 1 nor 2

Correct Answer : A

Answer Justification :

Conservation reserves and community reserves in India are terms denoting protected areas of India
which typically act as buffer zones to or connectors and migration corridors between established
national parks, wildlife sanctuaries and reserved and protected forests of India. Such areas are
designated as conservation areas if they are uninhabited and completely owned by the Government
of India but used for subsistence by communities and community areas if part of the lands are
privately owned.

These protected area categories were first introduced in the Wildlife (Protection)
Amendment Act of 2002 − the amendment to the Wildlife Protection Act of 1972. These
categories were added because of reduced protection in and around existing or proposed protected
areas due to private ownership of land, and land use. Hence, statement 1 is correct.

Meghalaya has the highest number of community reserves (65 ) amongst all the states. Kerala
has only one community reserve. Hence, statement 2 is incorrect

http://wiienvis.nic.in/Database/cri_8228.aspx

58 Consider the following statements:


1. Cold desert Biosphere Reserve is the largest Biosphere Reserve of India by area.
2. Tirupati is located in the Seshachalam Biosphere Reserve.
3. Banni Grasslands are found in the Kachch Biosphere Reserve.

Which of the statements given above is/are correct?


A. 1 only
B. 2 and 3 only
C. 1 and 3 only
D. 1, 2 and 3

39
Total Marks : 200
Online Prelims TEST - 12 (SUBJECT WISE)
( InsightsIAS Mock Test Series for UPSC Preliminary Exam 2020 )

Correct Answer : B

Answer Justification :

40
Total Marks : 200
Online Prelims TEST - 12 (SUBJECT WISE)
( InsightsIAS Mock Test Series for UPSC Preliminary Exam 2020 )

Kachch is the largest biosphere reserve in India. Hence, statement 1 is incorrect.

Tirupati is located in the Seshachalam Biosphere Reserve. Hence, statement 2 is correct.

Banni Grasslands Reserve or Banni grasslands form a belt of arid grassland ecosystem on
the outer southern edge of the desert of the marshy salt flats of Rann of Kutch in Kutch
District, Gujarat State, India. They are known for rich wildlife and biodiversity. Hence, statement
3 is correct.

59 What is/are objectives of the Convention on Biological Diversity?


1. Conservation of biodiversity
2. Sustainable use of biodiversity
3. Fair and equitable sharing of the benefits arising from the use of genetic resources.

Select the correct answer using the code given below:


A. 1 only
B. 2 and 3 only
C. 1 and 3 only
D. 1, 2 and 3

Correct Answer : D

Answer Justification :

41
Total Marks : 200
Online Prelims TEST - 12 (SUBJECT WISE)
( InsightsIAS Mock Test Series for UPSC Preliminary Exam 2020 )

All the statements given above are correct.

The Convention on Biological Diversity (CBD) is an international legally-binding treaty with


three main goals: conservation of biodiversity; sustainable use of biodiversity; fair and
equitable sharing of the benefits arising from the use of genetic resources. Its overall
objective is to encourage actions, which will lead to a sustainable future.

Hence, option (d) is correct.

60 Which of the following statements is/are correct regarding the Asiatic Lion species?
1. They are larger than African Lions.
2. It is listed in Appendix I of CITES.
3. It is categorized as near threatened on IUCN Red List.

Select the correct answer using the code given below:


A. 1 and 2 only
B. 2 only
C. 2 and 3 only
D. 1, 2 and 3

Correct Answer : B

Answer Justification :

Asiatic lions are slightly smaller than African lions. Adult males weigh 160 to 190 kg, while
females weigh 110 to 120 kg. The height at the shoulders is about 3.5 ft (110 cm). Hence,
statement 1 is incorrect.

The most striking morphological character, which is always seen in Asiatic lions, and rarely in
African lions, is a longitudinal fold of skin running along its belly.

At present Gir National Park and Wildlife Sanctuary is the only abode of the Asiatic lion.

It is listed in Schedule I of Wildlife (Protection) Act 1972, in Appendix I of CITES and as


Endangered on IUCN Red List. Hence, statement 2 is correct and statement 3 is incorrect.

https://www.wwfindia.org/about_wwf/priority_species/threatened_species/asiatic_lion/

61 Consider the following greenhouse gases:


1. Carbon Dioxide (CO2)
2. Nitrous oxide (N2O)
3. Sulfur Hexaflouride (SF6)
4. Methane (CH4)
5. Perfluorocarbons (PFCs);

Which of the above gases is/are placed under Kyoto Protocol?

42
Total Marks : 200
Online Prelims TEST - 12 (SUBJECT WISE)
( InsightsIAS Mock Test Series for UPSC Preliminary Exam 2020 )

A. 1, 2, 3 and 4 only
B. 1, 3, 4 and 5 only
C. 1 and 5 only
D. 1, 2, 3, 4 and 5

Correct Answer : D

Answer Justification :

The targets for the first commitment period of the Kyoto Protocol cover emissions of the six
main greenhouse gases, namely:

Carbon dioxide (CO2);


Methane (CH4);
Nitrous oxide (N2O);
Hydrofluorocarbons (HFCs);
Perfluorocarbons (PFCs); and
Sulphur hexafluoride (SF6)

Now, HFCs are placed under Montreal Protocol through Kigali Agreement.

62 Which of the following species is/ are considered critically endangered species in India as per IUCN
redlist?
1. Great Indian Bustard
2. Anaimalai Flying Frog
3. Ganges Shark

Select the correct answer using the code given below:


A. 1 and 2 only
B. 2 only
C. 1 and 3 only
D. 1, 2 and 3

Correct Answer : D

Answer Justification :

All the above given species are critically endangered.

Great Indian Bustard only occurs in the Thar desert of Indian Subcontinent and among the
heaviest of the flying birds in the world. Indian bustard is species is critically endangered because
of loss of its habitat and hunting.

43
Total Marks : 200
Online Prelims TEST - 12 (SUBJECT WISE)
( InsightsIAS Mock Test Series for UPSC Preliminary Exam 2020 )

Anaimalai Flying Frog, also known as false Malabar gliding frog, endemic to the Anaimalai Hills
of Western Ghats. Now these frogs have only been found in the Indira Gandhi National Park in
India.

Ganges River Shark is a species of requiem shark found in Brahmaputra River and Ganges River
of India. This species is one of the least known marine animal of India and confused with common
bull shark.

https://www.iucnredlist.org/species/22691932/134188105

https://www.iucnredlist.org/species/59016/11869234

https://www.iucnredlist.org/species/9281/12978210

63 Consider the following marine animals:


1. Sea Lion
2. Sea Horse
3. Sea Cow

Which of the above given marine animals is/are mammals?


A. 1 and 2 only
B. 2 only
C. 1 and 3 only
D. 1, 2 and 3

Correct Answer : C

Answer Justification :

Sea Lions are marine mammals classified as pinnipeds which means that they feature winged
feet. Sea Lions are found in bodies of water all over the world, but not in the Northern Atlantic
Ocean. Hence, statement 1 is correct.

Seahorse is the name given to 45 species of small marine fish in the genus Hippocampus.
Hence, statement 2 is incorrect.

Manatees are large, fully aquatic, mostly herbivorous marine mammals sometimes known as
sea cows. Hence, statement 3 is correct.

https://www.sealion-world.com

64 With reference to the Global Soil biodiversity Atlas, Consider the following statements:
1. It is a joint venture of the Global Soil Biodiversity Initiative and the European Commission Joint
Research Centre.
2. It is the first synthesis of global soil biodiversity research and its importance to our living world.

44
Total Marks : 200
Online Prelims TEST - 12 (SUBJECT WISE)
( InsightsIAS Mock Test Series for UPSC Preliminary Exam 2020 )

Which of the statements given above is/are correct ?


A. 1 only
B. 2 only
C. Both 1 and 2
D. Neither 1 nor 2

Correct Answer : C

Answer Justification :

Both statements are correct.

Global Soil biodiversity Atlas is a joint venture from the Global Soil Biodiversity Initiative and
the European Commission Joint Research Centre, the Global Soil Biodiversity Atlas is the
first synthesis of global soil biodiversity research and its importance to our living world.

https://www.globalsoilbiodiversity.org/atlas-introduction

65 Serengeti National Park , which was in the news recently, located in which of the following
countries?

A. Botswana
B. Tanzania
C. South Africa
D. Kenya

Correct Answer : B

Answer Justification :

Serengeti National Park, in northern Tanzania, is known for its massive annual migration of
wildebeest and zebra. Seeking new pasture, the herds move north from their breeding grounds in
the grassy southern plains. Many cross the marshy western corridor’s crocodile-infested Grumeti
River. Others veer northeast to the Lobo Hills, home to black eagles. Black rhinos inhabit the
granite outcrops of the Moru Kopjes.

Hence, option (b) is correct.

https://www.africa.com/serengeti-once-again-chosen-as-best-safari-park-of-africa/

66 Which of the following modification occurs/occur in mangrove forests as part of adaptation in saline
environment?
1. Laterally spreading roots
2. Vivipary mode of seed germination
3. Prop and Stilt roots

45
Total Marks : 200
Online Prelims TEST - 12 (SUBJECT WISE)
( InsightsIAS Mock Test Series for UPSC Preliminary Exam 2020 )

Select the correct answer using the code given below:


A. 1 and 2 only
B. 3 only
C. 2 and 3 only
D. 1, 2 and 3

Correct Answer : D

Answer Justification :

Mangrove trees have become specialized to survive in the extreme conditions of estuaries.
Two key adaptations they have are the ability to survive in waterlogged and anoxic (no oxygen)
soil, and the ability to tolerate brackish waters.

Some mangroves remove salt from brackish estuarine waters through ultra-filtration in their
roots. Other species have special glands on their leaves that actively secrete salt, a process that
leaves visible salt crystals on the upper surface of the leaves.

All mangrove species have laterally spreading roots with attached vertical anchor roots. These
roots are very shallow. Because the soil in shallow areas of mangal forests is typically flooded
during high tides, many species of mangrove trees have aerial roots, called pneumatophores,
which take up oxygen from the air for the roots. Some species also have prop roots or stilt roots
extending from the trunk or other roots that help them withstand the destructive action of tides,
waves, and storm surges (Smith and Smith, 2000).

Many mangrove trees also have a unique method of reproduction. Instead of forming seeds that fall
to the soil below and begin growing, mangrove seeds begin growing while still attached to the
parent plant. Vivipary mode of seed germination. These seedlings, called propagules, even grow
roots. After a period of growth, these seedlings drop to the water below and float upright until they
reach water that is shallow enough for their roots to take hold in the mud

https://oceanservice.noaa.gov/education/kits/estuaries/media/supp_estuar07d_mangrove.html

67 Which of the following organization publishes the ‘Global Environment Outlook’?

A. United Nations Framework Convention on Climate Change (UNFCCC)


B. United Nations Environment Programme (UNEP)
C. Food and Agriculture Organization (FAO)
D. United Nation Development Program (UNDP)

Correct Answer : B

Answer Justification :

Global Environment Outlook (GEO) is a series of reports on the environment issued periodically

46
Total Marks : 200
Online Prelims TEST - 12 (SUBJECT WISE)
( InsightsIAS Mock Test Series for UPSC Preliminary Exam 2020 )

by the United Nations Environment Programme (UNEP).

Hence, option (b) is correct.

https://www.unenvironment.org/resources/global-environment-outlook-6

68 Consider the following statements:


1. The Ocean Cleanup Project develops advanced technologies to rid the world's oceans of plastic.
2. The Great Pacific garbage patch is a gyre of marine debris particles in the south central Pacific
Ocean.

Which of the statements given above is/are correct?


A. 1 only
B. 2 only
C. Both 1 and 2
D. Neither 1 nor 2

Correct Answer : A

Answer Justification :

The Ocean Cleanup is a non-profit organization, developing advanced technologies to rid the
world’s oceans of plastic. Hence, statement 1 is correct.

By utilizing the ocean currents to the advantage, they have estimated to clean up half the Great
Pacific Garbage Patch in 5 years’ time.

The Great Pacific Garbage Patch (GPGP) is the largest of the five offshore plastic
accumulation zones in the world’s oceans. It is located halfway between Hawaii and
California. It is a gyre of marine debris particles in the north central Pacific Ocean. Hence,
statement 2 is incorrect.

https://theoceancleanup.com/great-pacific-garbage-patch/

69 With reference to the National Clean Air Programme (NCAP), consider the following statements:
1. The tentative national level target of 20%–30% reduction of PM2.5 and PM10 concentration by 2024
is proposed under the NCAP taking 2017 as the base year for the comparison of concentration.
2. The NCAP will be a mid-term, five-year action plan with 2019 as the first year.

Which of the statements given above is/are correct?


A. 1 only
B. 2 only
C. Both 1 and 2
D. Neither 1 nor 2

47
Total Marks : 200
Online Prelims TEST - 12 (SUBJECT WISE)
( InsightsIAS Mock Test Series for UPSC Preliminary Exam 2020 )

Correct Answer : C

Answer Justification :

Goal of NCAP is to meet the prescribed annual average ambient air quality standards at all locations
in the country in a stipulated timeframe. The tentative national level target of 20%–30%
reduction of PM2.5 and PM10 concentration by 2024 is proposed under the NCAP taking
2017 as the base year for the comparison of concentration. Hence, statement 1 is correct.

The NCAP will be a mid-term, five-year action plan with 2019 as the first year. Hence, statement 2
is correct.

http://vikaspedia.in/energy/policy-support/environment-1/air-1/national-clean-air-programme

70 With reference to Paris Climate Agreement, which of the following statements is/are correct?
1. Its long-term goal is to keep the increase in global average temperature to well below 2°C above
pre-industrial levels, and to pursue efforts to limit the increase to 1.5 °C.
2. The agreement is legally binding to member countries.
3. USA has announced its intention of withdrawal from the agreement.

Select the correct answer using the code given below.


A. 1 only
B. 1 and 3 only
C. 2 and 3 only
D. 1, 2 and 3

Correct Answer : B

Answer Justification :

The Paris Agreement is an agreement within the United Nations Framework Convention on Climate
Change, dealing with greenhouse-gas-emissions mitigation, adaptation, and finance, signed in 2016.

The Paris Agreement's long-term temperature goal is to keep the increase in global average
temperature to well below 2 °C above pre-industrial levels; and to pursue efforts to limit the
increase to 1.5 °C, recognizing that this would substantially reduce the risks and impacts of climate
change. Hence, statement 1 is correct.

In June 2017, U.S. President Donald Trump announced his intention to withdraw the United States
from the agreement. Hence, statement 3 is correct.

The Paris Agreement has a 'bottom up' structure in contrast to most international environmental
law treaties, which are 'top down', characterised by standards and targets set internationally, for
states to implement. Unlike its predecessor, the Kyoto Protocol, which sets commitment targets that
have legal force, the Paris Agreement, with its emphasis on consensus-building, allows for voluntary
and nationally determined targets. The specific climate goals are thus politically encouraged,
rather than legally bound. Hence, statement 2 is incorrect.

48
Total Marks : 200
Online Prelims TEST - 12 (SUBJECT WISE)
( InsightsIAS Mock Test Series for UPSC Preliminary Exam 2020 )

71 Consider the following protected areas:

Which of the above is/are declared Tiger Reserves?


A. Nagarahole
B. Indravati
C. Buxa
D. Palamau

Correct Answer : D

Answer Justification :

Project Tiger was launched by the Government of India in the year 1973 to save the endangered
2
species of tiger in the country. A total area of 71027.10 km is covered by these project tiger areas.

49
Total Marks : 200
Online Prelims TEST - 12 (SUBJECT WISE)
( InsightsIAS Mock Test Series for UPSC Preliminary Exam 2020 )

Hence, option (d) is correct.

72 Mankidia tribes, sometimes seen in news, are Particularly Vulnerable Tribal Group, in state of:

50
Total Marks : 200
Online Prelims TEST - 12 (SUBJECT WISE)
( InsightsIAS Mock Test Series for UPSC Preliminary Exam 2020 )

A. Himachal Pradesh
B. Maharashtra
C. Odisha
D. Kerala

Correct Answer : C

Answer Justification :

Mankidia, one of the 13 Particularly Vulnerable Tribal Groups (PVTG) in Odisha.

Mankidia, a marginalised group that critically depends on making rope with siali fibre that’s richly
available in Similipal.

https://www.thehindu.com/news/national/mankidia-denied-habitat-in-simlipal/article22392195.ece

73 Matters related to which of the following act can be adjudicated by the National Green Tribunal
(NGT)?
#00000
1. Wildlife (Protection) Act, 1972
2. The Biological Diversity Act, 2002
3. The Public Liability Insurance act, 1991

Select the correct answer using the code given below:


A. 2 only
B. 2 and 3 only
C. 1 and 3 only
D. 1, 2 and 3

Correct Answer : B

Answer Justification :

The National green tribunal (NGT) came into existence in 2010 under the
National Green Tribunal act 2010. The NGT act provides for the establishment of the National
green tribunal for the purpose of effective and quick disposal of various cases related to the
protection and conservation of forests, environment and other natural resources including the
enforcement of any legal right related to the environment.

The Tribunal shall not be bound by the procedure laid down under the Code of Civil Procedure,
1908, but shall be guided by principles of natural justice.

The jurisdiction of NGT extends over all the civil cases where there is a
substantial question of environment, and such questions arising out of the
implementation of enactments under schedule 1 of NGT act 2010. The schedule 1 of NGT includes
the following acts.

51
Total Marks : 200
Online Prelims TEST - 12 (SUBJECT WISE)
( InsightsIAS Mock Test Series for UPSC Preliminary Exam 2020 )

1. The Water (prevention and control of pollution) Act, 1974.


2. The Water (prevention and control of pollution) Cess Act, 1977.
3. The Forest (conservation) Act, 1980.
4. The Air (prevention and control of pollution) Act, 1981.
5. The Environment (Protection) Act, 1986.
6. The Public Liability Insurance Act, 1991.
7. The Biological Diversity Act, 2002.

Hence, option (b) is correct.

https://ngtonline.nic.in/newsite/about-us

74 Consider the following statements:


1. The Ministry of Earth Science represents India in the Belmont Forum.
2. Capacity Building Initiative for Transparency (CBIT) Fund is an outcome of the UNFCCC's Durban
agreement.
3. Kigali Agreement brought Hydrofluorocarbons (HFCs) under the Montreal protocol.

Which of the statements given above is/are correct?


A. 1 only
B. 2 and 3 only
C. 1 and 3 only
D. 1, 2 and 3

Correct Answer : C

Answer Justification :

Established in 2009, the Belmont Forum is a partnership of funding organizations, international


science councils, and regional consortia committed to the advancement of interdisciplinary and
trans disciplinary science.

The Ministry of Earth Science represents India in the Belmont Forum since 2012. Hence,
statement 1 is correct.

The Global Environment Facility (GEF) has set up a new financial initiative and trust fund to
help developing countries monitor and report the progress on their climate actions domestically.
GEF established the Capacity-Building Initiative for Transparency (CBIT) fund in response to
a decision during COP21 in Paris for an urgent reporting-related mechanism, including through
voluntary donor contributions. Hence, statement 2 is incorrect.

The Kigali Amendment to the Montreal Protocol on Substances that Deplete the Ozone
Layer entered into force on 1 January 2019, following ratification by 65 countries. The UN
Environment Programme (UNEP, or UN Environment) announced the entry into force, and noted
that it will help reduce the production and consumption of hydrofluorocarbons (HFCs), potent
greenhouse gases (GHGs), and thus to avoid global warming by up to 0.4°C this century. Hence,
statement 3 is correct.

52
Total Marks : 200
Online Prelims TEST - 12 (SUBJECT WISE)
( InsightsIAS Mock Test Series for UPSC Preliminary Exam 2020 )

http://www.belmontforum.org/about/

https://www.downtoearth.org.in/news/climate-change/new-climate-fund-established-to-help-developi
ng-countries-track-and-report-progress-on-climate-actions-54299

https://sdg.iisd.org/news/kigali-amendment-enters-into-force-bringing-promise-of-reduced-global-wa
rming/

75 With reference to the Green Climate Fund (GCF), consider the following statements:
1. The Green Climate Fund (GCF) is a fund established within the framework of the UNFCCC.
2. The Fund’s investments can be in the form of grants only.
3. It aims to catalyze a flow of climate finance to invest in low-emission and climate-resilient
development

Which of the statements given above is/are correct?


A. 1 and 2 only
B. 2 and 3 only
C. 1 and 3 only
D. 1, 2 and 3

Correct Answer : C

Answer Justification :

The Green Climate Fund (GCF) is a new global fund created to support the efforts of developing
countries to respond to the challenge of climate change. GCF helps developing countries limit or
reduce their greenhouse gas (GHG) emissions and adapt to climate change. It seeks to
promote a paradigm shift to low-emission and climate-resilient development, taking into
account the needs of nations that are particularly vulnerable to climate change impacts. Hence,
statement 3 is correct.

It was set up by the 194 countries who are parties to the United Nations Framework
Convention on Climate Change (UNFCCC) in 2010, as part of the Convention’s financial
mechanism. It aims to deliver equal amounts of funding to mitigation and adaptation, while being
guided by the Convention’s principles and provisions. Hence, statement 1 is correct.

The Fund pays particular attention to the needs of societies that are highly vulnerable to the effects
of climate change, in particular Least Developed Countries (LDCs), Small Island Developing States
(SIDS), and African States.

The Fund’s investments can be in the form of grants, loans, equity or guarantees. Hence,
statement 2 is incorrect.

https://www.greenclimate.fund/who-we-are/about-the-fund

76 The Talanoa Dialogue, sometime seen in news, is part of

53
Total Marks : 200
Online Prelims TEST - 12 (SUBJECT WISE)
( InsightsIAS Mock Test Series for UPSC Preliminary Exam 2020 )

A. IUCN
B. UNEP
C. CITES
D. UNFCCC

Correct Answer : D

Answer Justification :

The political phase of the Talanoa Dialogue ended at 24th Conference of Parties (COP 24)
to the United Nations Framework Convention on Climate Change (UNFCCC) in Katowice,
Poland, with a “call to action” issued jointly by Fijian (COP23) and the Polish Presidency.

The Talanoa Dialogue was convened as part of the UN climate talks. Talanoa is the Fijian traditional
way of holding conversations to tackle collective issues.

The objective of the dialogue is two-fold: to take stock of the progress of climate action since Paris
Agreement adoption and to inform the next round of Nationally Determined Contributions (NDCs)
with a view to raise climate ambition. The Talanoa platform was launched on January 10, 2018, and
since then it received 473 inputs, 44 from parties and 429 from non‐party stakeholders.

Under the Talanoa process, stakeholders talked about the three agreed guiding questions on the
climate crisis:

Where we are?
Where do we want to go?
How do we get there?

Hence, option (d) is correct.

https://www.downtoearth.org.in/news/climate-change/cop24-talanoa-dialogue-ends-with-a-weak-decl
aration-62468

77 With reference to the Urban Climate Change Resilience Trust Fund (UCCRTF), which of the following
statements is/are correct?
1. The UCCRTF was established under the Urban Financing Partnership Facility.
2. The fund is administered by World Bank.

Select the correct answer using the code given below:


A. 1 only
B. 2 only
C. Both 1 and 2
D. Neither 1 nor 2

Correct Answer : A

54
Total Marks : 200
Online Prelims TEST - 12 (SUBJECT WISE)
( InsightsIAS Mock Test Series for UPSC Preliminary Exam 2020 )

Answer Justification :

The Urban Climate Change Resilience Trust Fund is a $150 million multi-donor trust fund
(2013-2021) administered by Asian Development Bank (ADB) under the Urban Financing
Partnership Facility. Hence, statement 1 is correct and statement 2 is incorrect.

It aims to support fast-growing cities in Asia to reduce the risks poor and vulnerable people face
from floods, storms or droughts, by helping to better plan and design infrastructure to invest
against these impacts.

The fund supports cities by improving urban planning, designing climate resilient infrastructure and
investing in projects and people. The program’s goals include the roll out of about 25 infrastructure
projects and other resilience measures to protect around 2 million poor and vulnerable people in
the target cities by 2021. It also aims to leverage about $1 billion in investments from public,
private and municipal sources. The fund aims to scale up investments in urban climate change
resilience (UCCR), especially for the urban poor across 25 secondary cities in Asia and prioritizes
in eight developing member countries (DMCs) of the ADB: Bangladesh, India, Indonesia,
Myanmar, Nepal, Pakistan, the Philippines and Vietnam.

78 With reference to the Global Partnership on Forest and Landscape Restoration (GPFLR), consider the
following statements:
1. It is a proactive global network that unites governments, organizations, academic/research
institutes, communities and individuals.
2. It responds directly to the Bonn Challenge to restore 150 million hectares of deforested and
degraded land by 2030 and 350 million hectares by 2050.

Which of the statements given above is/are correct?


A. 1 only
B. 2 only
C. Both 1 and 2
D. Neither 1 nor 2

Correct Answer : A

Answer Justification :

The Global Partnership on Forest and Landscape Restoration (GPFLR) is a proactive global
network that unites governments, organizations, academic/research institutes,
communities and individuals under a common goal: to restore the world’s lost and degraded
forests and their surrounding landscapes. Hence, statement 1 is correct.

Specifically, the GPFLR responds directly to the Bonn Challenge to restore 150 million
hectares of deforested and degraded land by 2020 and 350 million hectares by 2030.
Hence, statement 2 is incorrect.

The GPFLR was initiated in 2003 by a small consortium of like-minded organizations and
spearheaded by IUCN. Its purpose is to catalyze dynamic, voluntary action through sharing diverse

55
Total Marks : 200
Online Prelims TEST - 12 (SUBJECT WISE)
( InsightsIAS Mock Test Series for UPSC Preliminary Exam 2020 )

experiences on restoration efforts which deliver tangible benefits to both local communities and
nature through a landscape approach, while also fulfilling international commitments on forests.

http://www.forestlandscaperestoration.org/why-gpflr

79 Which of the following statements are correct regarding ‘Sustainable Energy for All’ (SEforALL)?
1. It is an intergovernmental organization working with leaders in the government, the private sector
and the civil society to drive faster action toward the achievement of sustainable development.
2. It is a global initiative launched under the aegis of the United Nations Secretary General.
3. It is working with objective of faster action toward achievement of Sustainable Development Goal
7 (SDG7).

Select the correct answer using the code given below:


A. 2 and 3 only
B. 1 and 3 only
C. 1 and 2 only
D. 1, 2 and 3

Correct Answer : A

Answer Justification :

Sustainable Energy for All (SEforALL) is an international organization working with leaders in
government, the private sector and civil society to drive further, faster action toward
achievement of Sustainable Development Goal 7 (SDG7), which calls for universal access to
sustainable energy by 2030, and the Paris Agreement, which calls for reducing greenhouse gas
emissions to limit climate warming to below 2° Celsius. Hence, statement 3 is correct.

Former UN Secretary-General Ban Ki-moon launched the Sustainable Energy for All
initiative in 2011. Hence, statement 2 is correct.

Now an independent organization, it maintains close links with the UN, including through a
relationship agreement, partnerships with UN agencies. Hence, statement 1 is incorrect.

https://www.seforall.org/about-us

80 With reference to Critical Wildlife Habitats, consider the following statements:


1. They are declared by the central Government under Forest Rights Act, 2006.
2. The guidelines on Critical Wildlife Habitats to be prepared by Ministry of Tribal Affairs.
3. The Chief Minister of a state will notify an Expert Committee for the purpose of identification of
critical wildlife habitats (CWH) in a national park or sanctuary.

Which of the statements given above is/are correct?


A. 1 only
B. 2 and 3 only
C. 1 and 2 only

56
Total Marks : 200
Online Prelims TEST - 12 (SUBJECT WISE)
( InsightsIAS Mock Test Series for UPSC Preliminary Exam 2020 )

D. 1, 2 and 3

Correct Answer : A

Answer Justification :

Critical Wildlife Habitats (CWLHs) are such areas of the National Parks and Sanctuaries that are
required to be kept as ‘inviolate’ for the purpose of wildlife conservation as determined and notified
by the MoE&F, after an open process of consultation by an Expert Committee. Such areas are to be
clearly identified on case-to-case basis following scientific and objective criteria and only after
settling the rights of tribals and other traditional forest dwellers.

The guidelines on Critical Wildlife Habitats were prepared by MoEF&CC under Section 2 (b) of the
Scheduled Tribes and Other Traditional Forest Dwellers (Recognition of Forest Rights) Act, 2006
(FRA). Hence, statement 1 is correct.

The phrase ‘critical wildlife habitat’ is defined only in the Scheduled Tribes and Other Traditional
Forest Dwellers (Recognition of Forest Rights) Act, 2006, and NOT in the Wildlife (Protection) Act,
1972.

While MoTA is the nodal authority for FRA, the law identifies MoEF&CC as the agency to notify the
guidelines. Hence, statement 2 is incorrect.

In March 2017, the National Tiger Conservation Authority (NTCA) issued an order to deny
forest rights in critical tiger habitats (core areas of tiger reserves) in the absence of CWH
guidelines.

MoEF&CC finally issued CWH guidelines in January 2018.

The Chief Wildlife Warden of a state will notify an Expert Committee for the purpose of
identification of critical wildlife habitats (CWH) in a national park or sanctuary. Hence, statement
3 is incorrect.

https://www.downtoearth.org.in/news/governance/critical-wildlife-habitat-guidelines-issued-ntca-ord
er-superseded-59934

https://pib.gov.in/newsite/PrintRelease.aspx?relid=69806

81 With reference to the Citizenship (Amendment) Bill, 2019, consider the following statements:
1. The Bill amends the Citizenship Act, 1955 to make illegal migrants who are Hindus, Sikhs,
Buddhists, Jains, Parsis, Christians, Jews and atheists from Afghanistan, Bangladesh and Pakistan,
eligible for citizenship.
2. The Bill has been lapsed as it could not be passed in the Lok sabha.

Which of the statements given above is/are correct?

57
Total Marks : 200
Online Prelims TEST - 12 (SUBJECT WISE)
( InsightsIAS Mock Test Series for UPSC Preliminary Exam 2020 )

A. 1 only
B. 2 only
C. Both 1 and 2
D. Neither 1 nor 2

Correct Answer : D

Answer Justification :

Citizenship (Amendment) Bill, 2019 makes an amendment to the Citizenship Act, 1955, the umbrella
law that sets out the elements of Indian citizenship. The Amendment stipulates that “persons
belonging to minority communities, namely, Hindus, Sikhs, Buddhists, Jains, Parsis and
Christians from Afghanistan, Bangladesh and Pakistan shall not be treated as illegal migrants for
the purposes of that Act”. It do not include Jews and atheists.

Hence, statement 1 is incorrect.

The Bill has been passed by Lok Sabha. However, it has been lapsed as it could not be passed in the
Rajya sabha. Hence, statement 2 is incorrect.

https://www.thehindu.com/opinion/lead/a-bill-that-undercuts-key-constitutional-values/article296117
70.ece

82 In which of the following towns, the famous open rock relief the Descent of the Ganges, also known
as Arjuna's Penance is located?

A. Mamallapuram
B. Ajanta
C. Kanchipuram
D. Hampi

Correct Answer : A

Answer Justification :

Mamallapuram, also called Mahabalipuram or Seven Pagodas, historic town, northeast Tamil
Nadu state, southeastern India. It lies along the Coromandel Coast of the Bay of Bengal.

The town’s religious centre was founded by a 7th-century-


CE Hindu Pallava king—Narasimhavarman, also known as Mamalla—for whom the town was
named.

Ancient Chinese, Persian, and Roman coins found at Mamallapuram point to its earlier existence as
a seaport.

It contains many surviving 7th- and 8th-century Pallava temples and monuments, chief of which are

58
Total Marks : 200
Online Prelims TEST - 12 (SUBJECT WISE)
( InsightsIAS Mock Test Series for UPSC Preliminary Exam 2020 )

the sculptured rock relief popularly known as “Arjuna’s Penance,” or “Descent of the
Ganges,” a series of sculptured cave temples, and a Shaiva temple on the seashore. The
town’s five rathas, or monolithic temples, are the remnants of seven temples, for which the town
was known as Seven Pagodas. The entire assemblage collectively was designated a
UNESCO World Heritage site in 1984.

Hence, option (a) is correct.

https://www.thehindu.com/opinion/lead/in-search-of-the-wuhan-spirit/article29598178.ece

https://www.britannica.com/place/Mamallapuram

83 Consider the following statements:


1. U.A.E is India’s second largest goods trading partner after China in the financial year 2019.
2. In 2018, India and China launched joint programme to train Afghan diplomats.
3. Doklam plateau is a disputed territory between India and China.

Which of the statements given above is/are correct?


A. 2 only
B. 1 and 3 only
C. 2 and 3 only
D. 1, 2 and 3

Correct Answer : A

Answer Justification :

Even as trade war begins between two countries, the US has pipped China to become India’s
top goods trade partner in FY19, the official data showed. In FY18, China was India’s largest
trade partner.

The other top ten trading partners on the list include the UAE, Saudi Arabia, Hong Kong,
Singapore, Germany, Republic of Korea and Indonesia.

Hence, statement 1 is incorrect.

India and China have launched their first joint programme for Afghanistan to train its
diplomats. Hence, statement 2 is correct.

Doklam, is an area with a plateau and a valley, lying between China's Chumbi Valley to the north,
Bhutan's Ha Valley to the east and India's Sikkim state to the west. It has been depicted as part
of Bhutan in the Bhutanese maps since 1961, but it is also claimed by China. Hence,
statement 3 is incorrect.

59
Total Marks : 200
Online Prelims TEST - 12 (SUBJECT WISE)
( InsightsIAS Mock Test Series for UPSC Preliminary Exam 2020 )

https://www.financialexpress.com/economy/us-pips-china-to-become-indias-largest-trade-partner-am
id-tariff-war-check-others-in-top-10/1611899/

https://www.thehindubusinessline.com/news/world/india-china-launch-joint-programme-to-train-afgh
an-diplomats/article25216334.ece

84 The One Stop Wedding Shop In Mysuru!


1. The state co-operative banks do not lend the money directly to the farmers.
2. The Land development banks do not accept deposits from the general public.
3. Urban Co-operative Banks (UCBs) are allowed to lend money for both agricultural and non-
agricultural purposes.

Which of the statements given above is/are correct?


A. 3 only
B. 1 and 2 only
C. 2 and 3 only
D. 1, 2 and 3

Correct Answer : D

Answer Justification :

The state co-operative bank is a federation of central co-operative bank and acts as a watchdog of
the co-operative banking structure in the state.

The state co-operative banks lend money to central co-operative banks and primary
societies and not directly to the farmers. Hence, statement 1 is correct.

The Land development banks are organized in 3 tiers namely; state, central, and primary level
and they meet the long term credit requirements of the farmers for developmental
purposes. Recently, the supervision of land development banks has been assumed by National Bank
for Agriculture and Rural development (NABARD). These banks do not accept deposits from the
general public. Hence, statement 2 is correct.

Urban Co-operative Banks (UCBs) till 1996, were allowed to lend money only for non-agricultural
purposes. This distinction does not hold today. Hence, statement 3 is correct.

https://www.thehindu.com/opinion/op-ed/co-operative-banks-is-dual-regulation-the-problem/article2
60
Total Marks : 200
Online Prelims TEST - 12 (SUBJECT WISE)
( InsightsIAS Mock Test Series for UPSC Preliminary Exam 2020 )

9587375.ece

85 Consider the following statements:


1. India shares the longest land boundary with Bangladesh.
2. India has trade deficit with Bangladesh.

Which of the statements given above is/are correct?


A. 1 only
B. 2 only
C. Both 1 and 2
D. Neither 1 nor 2

Correct Answer : A

Answer Justification :

Currently, India shares the longest land border with Bangladesh, stretching over 4,096.9
km. This boundary falls in five Indian states—West Bengal, Assam, Meghalaya, Mizoram and
Tripura. Hence, statement 1 is correct.

The Federation of Bangladesh Chambers of Commerce and Industries (FBCCI) Monday said
bilateral trade between Bangladesh & India was $9.5 billion in FY2017-18 with exports to
India pegged at $0.87 billion and imports, at $8.6 billion. Hence, statement 2 is incorrect.

https://www.thehindu.com/opinion/op-ed/furthering-this-neighbourhood-friendship/article29577509.
ece

https://www.orfonline.org/research/integrated-check-posts-on-the-india-bangladesh-border-a-field-su
rvey-and-brief-analysis-54559/

https://economictimes.indiatimes.com/news/economy/foreign-trade/bangladesh-india-bilateral-trade-
was-9-5-billion-in-fy2017-18-fbcci-president-sheikh-fazle-
fahim/articleshow/70229327.cms?from=mdr

86 Forest-PLUS 2.0, sometimes seen in news is recently launched by:

A. Greenpeace
B. United Nation Environment Program (UNEP)
C. US Agency for International Development (USAID) and India’s Ministry of Environment,
Forest and Climate Change (MoEF&CC)
D. Bombay Natural History Society (BNHS)

Correct Answer : C

Answer Justification :

61
Total Marks : 200
Online Prelims TEST - 12 (SUBJECT WISE)
( InsightsIAS Mock Test Series for UPSC Preliminary Exam 2020 )

US Agency for International Development (USAID) and India's Ministry of Environment,


Forest and Climate Change (MoEF&CC) officially launched Forest-PLUS 2.0 on September
25, 2019.

It is a five-year programme initiated in December 2018 that focuses on developing tools and
techniques to bolster ecosystem management and harnessing ecosystem services in forest
landscape management.

Forest-PLUS 2.0, the second set of pilot projects, is meant to enhance sustainable forest
landscape management after Forest-PLUS completed its five years in 2017.

The programme’s first set focused on capacity building to help India participate in Reducing
Emissions from Deforestation and forest Degradation (REDD+). It included four pilot projects
in Sikkim, Rampur, Shivamogga and Hoshangabad.

Hence, option (c) is correct.

https://www.downtoearth.org.in/news/forests/centre-us-agency-launch-forest-plus-2-0-66951

87 With reference to the National Population Register (NPR), consider the following statements:
1. It is a register of usual residents of the country.
2. It is not mandatory for every usual resident of India to register in the NPR.
3. The register would contain demographic as well as biometric particulars.

Which of the statements given above is/are correct?


A. 2 only
B. 1 and 3 only
C. 2 and 3 only
D. 1, 2 and 3

Correct Answer : B

Answer Justification :

The National Population Register (NPR) is a Register of usual residents of the country.
Hence, statement 1 is correct.

It is being prepared at the local (Village/sub-Town), sub-District, District, State and National level
under provisions of the Citizenship Act 1955 and the Citizenship (Registration of Citizens and issue
of National Identity Cards) Rules, 2003.

It is mandatory for every usual resident of India to register in the NPR. Hence, statement
2 is incorrect.

A usual resident is defined for the purposes of NPR as a person who has resided in a local area for
the past 6 months or more or a person who intends to reside in that area for the next 6 months or
more.

62
Total Marks : 200
Online Prelims TEST - 12 (SUBJECT WISE)
( InsightsIAS Mock Test Series for UPSC Preliminary Exam 2020 )

The objective of the NPR is to create a comprehensive identity database of every usual resident in
the country. The database would contain demographic as well as biometric particulars.
Hence, statement 3 is correct

https://indianexpress.com/article/explained/simply-put-listing-indias-residents-citizens-npr
-census-nrc-6032093/

http://censusindia.gov.in/2011-Common/IntroductionToNpr.html

88 Sendai Framework, sometimes seen in news, is related to?

A. Organic Farming
B. Climate Change
C. Forced Migration
D. Disaster Management

Correct Answer : D

Answer Justification :

The Sendai Framework for Disaster Risk Reduction 2015-2030 (Sendai Framework) is the
first major agreement of the post-2015 development agenda, with seven targets and four priorities
for action. It was endorsed by the UN General Assembly following the 2015 Third UN World
Conference on Disaster Risk Reduction (WCDRR).

Hence, option (d) is correct.

https://www.unisdr.org/we/coordinate/sendai-framework

89 Jamadani, sometimes seen in news, is related to:

A. Micro irrigation practice in Rajasthan


B. Weaving technique
C. Folk dance
D. Organic farming

Correct Answer : B

Answer Justification :

Jamdani is a fine muslin textile of Bengal, produced for centuries in South Rupshi of
Narayanganj district in Bangladesh. The historic production of jamdani was patronized by imperial
warrants of the Mughal emperors.

https://www.thehindu.com/life-and-style/fashion/textile-designer-gaurang-shah-to-unveil-khadi-saris-

63
Total Marks : 200
Online Prelims TEST - 12 (SUBJECT WISE)
( InsightsIAS Mock Test Series for UPSC Preliminary Exam 2020 )

with-a-woven-replication-of-raja-ravi-varma-paintings-at-ngma-mumbai-to-mark-gandhis-150th-birth-
anniversary/article29499710.ece

90 With reference to the Intergovernmental Panel on Climate Change (IPCC), consider the following
statements:
1. It was created in 1988 by the World Meteorological Organization(WMO) and the United Nations
Environment Programme (UNEP).
2. The IPCC does not conduct its own research.

Which of the statements given above is/are correct?


A. 1 only
B. 2 only
C. Both 1 and 2
D. Neither 1 nor 2

Correct Answer : C

Answer Justification :

The Intergovernmental Panel on Climate Change (IPCC) is the United Nations body for assessing
the science related to climate change. It was created in 1988 by the World Meteorological
Organization(WMO) and the United Nations Environment Programme (UNEP), the objective
of the IPCC is to provide governments at all levels with scientific information that they can use to
develop climate policies.

Hence, statement 1 is correct.

Through its assessments, the IPCC identifies the strength of scientific agreement in different areas
and indicates where further research is needed. The IPCC does not conduct its own research.

Hence, statement 2 is correct.

https://indianexpress.com/article/explained/why-latest-ipcc-report-matters-climate-change-report-60
35008/

https://www.ipcc.ch/about/

91 With reference to the India TB Report 2019, consider the following statements:
1. TB burden in India is highest in Maharashtra, followed by Uttar Pradesh.
2. The highest percentage of patients who tested positive for TB and were also infected with HIV came
from Assam.

Which of the statements given above is/are correct?


A. 1 only
B. 2 only
C. Both 1 and 2

64
Total Marks : 200
Online Prelims TEST - 12 (SUBJECT WISE)
( InsightsIAS Mock Test Series for UPSC Preliminary Exam 2020 )

D. Neither 1 nor 2

Correct Answer : D

Answer Justification :

The India TB Report 2019, was released by the government on September 25, 2019.

India ranks second in the world as far as TB-related mortality is concerned. About nine per
cent of the global burden of HIV-associated TB is borne by India.

Overall, the TB burden in India is highest in Uttar Pradesh. India’s largest state, accounting
for 17 per cent the country’s population, contributed 20 per cent of total TB notified patients.
Hence, statement 1 is incorrect.

The highest percentage of patients who tested positive for TB and were also infected with HIV came
from Nagaland (15.6 per cent), followed by Karnataka (10 per cent), Chandigarh (9.1 per
cent) and Manipur (8.9 per cent). Hence, statement 2 is incorrect.

https://www.downtoearth.org.in/news/health/hiv-associated-tuberculosis-on-the-rise-india-tb-report-
6693

92 Consider the following statements:


1. The religious book of the Satnamis is called Pothi.
2. Satnami rebellion was fought against the Mughal Empire in the Bengal province.
3. Aurangzeb crushed the Satnami rebellion.

Which of the statements given above is/are correct?


A. 3 only
B. 1 and 3 only
C. 1, 2 and 3
D. None

Correct Answer : B

Answer Justification :

The religious book of the Satnamis is called Pothi. Hence, statement 1 is correct.

The earliest Satnamis were a sect of mendicants and householders founded by Birbhan in
Narnaul in eastern Punjab in 1657. In 1672 they defied the Mughal emperor Aurangzeb and
were crushed by his army. Hence, statement 2 is correct and statement 3 is incorrect.

https://www.britannica.com/topic/Satnami-sect

93 Consider the following statements regarding Rural Sanitation Strategy (2019-2029):

65
Total Marks : 200
Online Prelims TEST - 12 (SUBJECT WISE)
( InsightsIAS Mock Test Series for UPSC Preliminary Exam 2020 )

1. It was launched by the Department of Drinking Water and Sanitation (DDWS), Ministry of Jal
Shakti.
2. It highlights innovative models for sanitation financing.

Which of the statements given above is/are correct?


A. 1 only
B. 2 only
C. Both 1 and 2
D. Neither 1 nor 2

Correct Answer : C

Answer Justification :

The Department of Drinking Water and Sanitation (DDWS), Ministry of Jal Shakti, GoI
launched the 10 Year Rural Sanitation Strategy (2019-2029), which focus on sustaining the
sanitation behavior change that has been achieved under the Swachh Bharat Mission Grameen
(SBM-G), ensuring that no one is left behind, and increasing access to solid and liquid waste
management. Hence, statement 1 is correct.

The 10 year strategy focuses on the need for States/UTs to continue their efforts to sustain the
gains of the mission through capacity strengthening, IEC (Information, education and
communication), organic waste management, plastic waste management, grey water management
and black water management. The strategy also speaks about potential collaborations with
development partners, civil society and inter-government partnerships. It also highlights
innovative models for sanitation financing. Hence, statement 2 is correct.

http://vikaspedia.in/news/national-launch-of-10-year-rural-sanitation-strategy

94 Consider the following statements regarding Rafael fighter Jet:


1. It is a multi-role combat fighter jet.
2. It is capable of carrying small nuclear weapons.
3. It can used for ground support, in-depth strike and anti-ship strike.

Which of the statements given above is/are correct?


A. 1 and 2 only
B. 1 and 3 only
C. 2 and 3 only
D. 1, 2 and 3

Correct Answer : D

Answer Justification :

The Dassault is a French twin-engine, canard delta wing, multirole fighter aircraft designed and
built by Dassault Aviation. Hence, statement 1 is correct.

66
Total Marks : 200
Online Prelims TEST - 12 (SUBJECT WISE)
( InsightsIAS Mock Test Series for UPSC Preliminary Exam 2020 )

Equipped with a wide range of weapons, the Rafale is intended to perform air
supremacy, interdiction, aerial reconnaissance, ground support, in-depth strike, anti-ship
strike and nuclear deterrence missions. Hence, both statement 2 and statement 3 are
correct.

https://www.news18.com/news/auto/with-rafale-fighter-jet-in-its-arsenal-iaf-firepower-has-grown-exp
onentially-heres-why-2333131.htm

95 Consider the following statements regarding INS Khanderi:


1. It belongs to the Kalvari class submarines.
2. It is indigenously built by the Indian Navy.
3. Recently, it was commissioned to Indian Navy.

Which of the statements given above is/are correct?


A. 1 and 2 only
B. 1 and 3 only
C. 2 and 3 only
D. 1, 2 and 3

Correct Answer : D

Answer Justification :

INS Khanderi (2017) is the second of the Indian Navy's six Kalvari-class submarines being
built in India. Hence, statement 1 is correct.

It is a diesel-electric attack submarine which was designed by French naval defence and energy
company DCNS and manufactured at Mazagon Dock Limited in Mumbai. Hence, statement 2
is correct.

Recently, it was commissioned by the Defence minister Rajnath Singh to Indian Navy. Hence,

67
Total Marks : 200
Online Prelims TEST - 12 (SUBJECT WISE)
( InsightsIAS Mock Test Series for UPSC Preliminary Exam 2020 )

statement 3 is correct.

https://www.indiatoday.in/mail-today/story/navy-set-to-induct-submarine-ins-khanderi-today-160406
7-2019-09-28

https://economictimes.indiatimes.com/news/defence/rajnath-singh-commissions-ins-khanderi-attack-
submarine/articleshow/71345859.cms

96 Which of the following organizations prepared the School Education Quality Index (SEQI) ?

A. Ministry Of Human Resource and Development


B. PRATHAM
C. NITI Aayog
D. UNESCO

Correct Answer : C

Answer Justification :

http://social.niti.gov.in/education-index

NITI Aayog has prepared the School Education Quality Index (SEQI) based on the data of
2016-17. The index aims to bring an outcomes focus to education policy by providing States and
UTs with a platform to identify their strengths and weaknesses and undertake requisite course
corrections or policy interventions.

Hence, option (c) is correct.

97 With reference to International Labour organization (ILO), consider the following statements:
1. It was created as part of the Treaty of Versailles.
2. The ILO became the first specialized agency of the UN in 1946.

Which of the statements given above is/are correct?


A. 1 only
B. 2 only
C. Both 1 and 2
D. Neither 1 nor 2

Correct Answer : C

Answer Justification :

As the ILO celebrates its 100th anniversary in 2019, it is timely to reflect on the many life-changing
events which are linked to the ten decades of ILO history.

68
Total Marks : 200
Online Prelims TEST - 12 (SUBJECT WISE)
( InsightsIAS Mock Test Series for UPSC Preliminary Exam 2020 )

The Organization has played a role at key historical junctures – the Great Depression,
decolonization, the creation of Solidarność in Poland, the victory over apartheid in South Africa –
and today in the building of an ethical and productive framework for a fair globalization.

It was created in 1919, as part of the Treaty of Versailles that ended World War I, to reflect
the belief that universal and lasting peace can be accomplished only if it is based on social justice.
Hence, statement 1 is correct.

The Constitution of the ILO was drafted in early 1919 by the Labour Commission, chaired
by Samuel Gompers, head of the American Federation of Labour (AFL) in the United States.

In 1946, the ILO became a specialized agency of the newly formed United Nations. Hence,
statement 2 is correct.

https://www.ilo.org/global/about-the-ilo/history/lang--en/index.htm

98 Which of the following organizations releases the Global Competitive Index?

A. IMF
B. World Bank
C. World Trade Organization
D. World Economic Forum (WEF)

Correct Answer : D

Answer Justification :

The Global Competitiveness Report (GCR) is a yearly report published by the World Economic
Forum (WEF). The Global Competitiveness Index measures the set of institutions, policies, and
factors that set the sustainable current and medium-term levels of economic prosperity.

Hence, option (d) is correct.

https://economictimes.indiatimes.com/news/economy/indicators/india-slips-10-places-on-global-com
petitiveness-index-singapore-on-top/articleshow/71498559.cms

99 With reference to Lal Bahadur Shastri, consider the following statements:


1. He had been jailed for participating in Individual Satyagraha.
2. He gave the slogan “ Jai Jawan, Jai Kisan”.
3. Under his leadership, India won the 1965 war against Pakistan.

Which of the statements given above is/are correct?


A. 1 and 2 only
B. 1 and 3 only
C. 2 and 3 only
D. 1, 2 and 3

69
Total Marks : 200
Online Prelims TEST - 12 (SUBJECT WISE)
( InsightsIAS Mock Test Series for UPSC Preliminary Exam 2020 )

Correct Answer : D

Answer Justification :

Lal Bahadur Shastri was the second Prime Minister of India and a senior leader of the Indian
National Congress political party. He joined the Indian independence movement in the 1920s.
Deeply impressed and influenced by Mahatma Gandhi.

In 1940, he was sent to prison for one year, for offering individual Satyagraha support to the
independence movement. Hence, statement 1 is correct.

He led the country during the Indo-Pakistan War of 1965. Hence, statement 3 is correct.

His slogan of "Jai Jawan Jai Kisan" ("Hail the soldier, Hail the farmer") became very popular
during the war. The war formally ended with the Tashkent Agreement on 10 January 1966. Hence,
statement 2 is correct.

http://www.newsonair.com/Main-News-Details.aspx?id=37237

100 In which of the following Indian cities, India’s first e-waste clinic to be set up?

A. Varanasi
B. Mysore
C. Bhopal
D. Hyderabad

Correct Answer : C

Answer Justification :

The Bhopal Municipal Corporation (BMC) and the Central Pollution Control Board (CPCB)
have joined hands to set up the country’s first e-waste clinic in Bhopal that would enable
segregation, processing and disposal of waste from both household and commercial units.

Hence, option (c) is correct.

https://www.thehindu.com/news/national/other-states/indias-first-e-waste-clinic-to-be-set-up-in-bhop
al/article29598165.ece

70

You might also like